Наука


Ответить в тред Ответить в тред

<<
Назад | Вниз | Каталог | Обновить тред | Автообновление
522 56 192

Задаём здесь умные вопросы Аноним 07/12/18 Птн 20:33:31 4543561
15343587230390.jpg (212Кб, 1218x1015)
1218x1015
Получаем тупые ответы
Аноним 07/12/18 Птн 20:40:57 4543572
Объясните пожалуйста движение колеса без проскальзывания. Там есть сила трения, но её момент равен нулю из-за того, что точка нижняя движется, и работы сила не совершает, внизу перемещения нет. Всё заебись, колесо может так двигаться бесконечно. Но что нам мешает записать второй закон Ньютона? Но тогда на колесо будет действовать ускорение. Так вот, как быть со вторым законом Ньютона?
Аноним 07/12/18 Птн 22:09:27 4543593
>>454357
> Там есть сила трения,
Ебанулся?
> точка нижняя движется
Упоролся?
Аноним 08/12/18 Суб 00:17:49 4543664
>>454357
Так если трения не будет, колесо не поедет.
А внизу точка не движется, конечно.
Аноним 08/12/18 Суб 00:18:46 4543675
Аноним 08/12/18 Суб 01:10:32 4543716
>>454366
Если колесо уже едет с постоянной скоростью, то силы трения нет. Где вопрос?
Аноним 08/12/18 Суб 09:25:09 4543767
Test
Аноним 08/12/18 Суб 10:22:24 4543788
>>454356 (OP)
Предположим, как то изъебнувшись, используя например "кошмарное дальнодействие" кто-то смог передать инфу быстрее скорости света. В чем парадокс того, что я узнал о событии раньше, чем свет от этого события дошел до меня.
Аноним 08/12/18 Суб 11:05:42 4543799
>>454378
Не предположим, корреляция состояний не является физической передачей информации.
И принцип причинности ввели для того, чтобы поля не "саморазрушались". Ну не работают иначе, епта. Т.е. без разрыва (нарушение его гладкости) поля нельзя послать сигнал быстрее света. А как разорвешь поле, то полезут демоны из варпа неустранимые бесконечности.
Аноним 08/12/18 Суб 15:02:26 45438710
Аноним 08/12/18 Суб 19:52:55 45440011
Если тёмная материя участвует в гравитационном взаимодействии, то почему она всегда остаётся "рыхлой", а не сбивается в сверхплотные комки?
Аноним 08/12/18 Суб 21:17:07 45440212
>>454400
Они и так сбиваются в большие тучи, называемые темным галом, и более того они образуют галактические нити.
Впрочем из-за то, что они не взаимодействуют ни с "обычной" материей (электромагнитно, и возможно так же в слабом взаимодействие, но поскольку эти ваши вимпы не обнаружены, они идут нахуй), ни сами с собой, то они не могут передать часть энергии-импульса излучению и следовательно сконденсироваться в более плотные структуры как "обычная" материя. Вот и получаем, что из-за невозможности "остыть" и упасть в гравитационную яму, они кружат вокруг нее.
Ну еще темная материя довольно тяжелая и "холодная" прям как твоя мамаша шлюха, это еще сильнее затрудняет кучкование.
Аноним 08/12/18 Суб 21:40:05 45440313
>>454402
>Впрочем из-за то, что они не взаимодействуют ни с "обычной" материей (электромагнитно, и возможно так же в слабом взаимодействие, но поскольку эти ваши вимпы не обнаружены, они идут нахуй), ни сами с собой, то они не могут передать часть энергии-импульса излучению и следовательно сконденсироваться в более плотные структуры как "обычная" материя. Вот и получаем, что из-за невозможности "остыть" и упасть в гравитационную яму, они кружат вокруг нее.
>Ну еще темная материя довольно тяжелая и "холодная" прям как твоя мамаша шлюха, это еще сильнее затрудняет кучкование.
Ты похоже туповат и не видишь, что высрав тут столько текста так и не написал что мешает ей кучковаться дальше
>Они и так сбиваются в большие тучи, называемые темным галом, и более того они образуют галактические нити.
Аноним 08/12/18 Суб 21:45:27 45440414
>>454403
>>454402
Хм, хотя возможно и написал. Мне нужно это обдумать.
Спасибо.
Аноним 09/12/18 Вск 00:09:17 45441115
Что такое сознание?
Аноним 09/12/18 Вск 00:28:09 45441216
>>454356 (OP)
Что если негр выебет альбиноса-европейца? Кто родится?
А если белый отъебет негра-альбиноса?
Аноним 09/12/18 Вск 02:39:21 45441317
b917e164e21efdb[...].jpg (147Кб, 2048x1365)
2048x1365
>>454412
>Что если негр выебет альбиноса-европейца? Кто родится?
>А если белый отъебет негра-альбиноса?
В любом случае польётся БЕЛАЯ малафья из жопы.
Аноним 09/12/18 Вск 09:31:52 45442018
76.jpg (15Кб, 207x269)
207x269
Аноним 09/12/18 Вск 09:44:12 45442319
unnamed.jpg (667Кб, 2448x2713)
2448x2713
Чё за растение?
Аноним 09/12/18 Вск 19:12:37 45447420
Бытовой вопрос на тему формальной логики. Я проходил в универе логику высказываний и предикатов, но, читая вики, обнаружил что, то что я делал в универе это дедуктивный Hilbert-style вывод, а есть ещё множестов других, натуральная дедукция, Gentzen style, sequent calculus. Эти системы предполагают разные аксиомы и разные правила вывода. А кроме того есть ещё какие-то неклассические логики. И всё это выводчисто дедуктивный, а индукция это вообще нейросети и машоб. Вопрос: откуда взялось такое многообразие логик ? Я думал что они возникают под задачу, но адекватных примеров задач на вики я не встретил ? Надо полагать что логики борются с чисто формальными задачами из сферы математики, а не с парадоксами на естественном языке как, например, парадокс лжеца ? Тогда на сколько это близко к сфере ИИ(а именно чат-ботам), т. е. с анализом естесттвенного языка и логики бытовой названные мной разделы логики отношения не имеют ? Около года назад на доске был интересный редпро ИИ, там человек сказал что натуральная дедукция ближе к человеческой лоике чем Hilbert-style. Это он тральнул тип, на самом деле это вообще области не пересекающиеся ? Стоит ли перед формальной символической логикой вообще задача аппроксимации человеческого мышления ?
https://en.wikipedia.org/wiki/Sequent_calculus#Distinction_between_natural_deduction_and_sequent_calculus
Аноним 09/12/18 Вск 20:29:13 45448521
>>454420
Тоже такое впечатление, но оно и понятно, ведь существует группа людей под названием "атеисты" и им как-то нужно оправдывать свою веру - вот они и оправдывают такой никудышной аргументацией.
Аноним 09/12/18 Вск 22:29:27 45449022
>>454485
Ну я как бы тоже не считаю, что для существования вселенной нужно 3е лицо в виде бога. Но ведь что - то же переживает опыт, и цветность вроде как тоже есть, но даже если это каким-то образом иллюзия и даже то что кто-то переживает эту иллюзию - тоже иллюзия, то аргументация все равно мне не понятна.
Аноним 09/12/18 Вск 23:26:59 45449323
Алло связь, хотел выебнуться, объяснив что гигантизм в мезозое был из за высокой плотности атмосферы, мол внешнее давление выравнивает внутреннее и растут гиганты, но меня обдристали всем тредом, мол это бред. Чё, рили бред? Палеонтологи на месте?
Аноним 10/12/18 Пнд 01:42:43 45450024
>>454493
mgh тебя в помощь, обдристанный
Аноним 10/12/18 Пнд 01:48:19 45450225
>>454493
>Алло связь, хотел выебнуться, объяснив что гигантизм в мезозое был из за высокой плотности атмосферы, мол внешнее давление выравнивает внутреннее и растут гиганты, но меня обдристали всем тредом, мол это бред.
Ты можешь просто убедить себя что ты зашёл туда всех толсто потроллить, и после того как спизданул ту хуйню это не тебя обдристали, а ты всех толсто затроллил. Победа.
Аноним 10/12/18 Пнд 04:43:20 45450826
>>454502
Да ты сам обдристан своим отсутствием знаний, по-любому у тебя все стены портретами Петрика увешаны и иконами, воскуряешь своей тупости ладан ежедневно, ведь в твоих словах логики вообще ноль, то же мне, придумал тут разоблачение, да засунь в жопу себе свою нелепую конструкцию, все равно она из соплей и говна, толсто потроллить, победа у него, блядь, да ты ничто вообще, кварк ты понял, струна, тебя вообще нет, как этих ваших выдуманных квантов
Аноним 10/12/18 Пнд 10:32:11 45452327
>>454505
Сычев, проснись
you obosralsya
Аноним 10/12/18 Пнд 10:39:44 45452528
>>454505
Я подобной херней страдал, когда мне было 9
Сколько тебе годиков, мой малыш?..
Аноним 10/12/18 Пнд 11:20:27 45452629
>>454508
>да ты ничто вообще, кварк ты понял, струна, тебя вообще нет
Если меня нет то чего ты тогда так порвался?)
Аноним 10/12/18 Пнд 11:25:23 45452730
Поцаны, объясните за зонды и прочие машины, отправляющиеся в космос? Каким образом они передают сигналы на землю? Что такое вообще наш вай-фай и радио?
Аноним 10/12/18 Пнд 11:27:38 45452831
>>454526
Ты порвался чего ты так порвался?) расскажи тогда порвался чего ты так, порвался?) а меня нет, а ты порвался) чего порвался, если нет?) так порвался, бомбит, баттхерт чего так взорвался бабах чего так?) порвался?))
Аноним 10/12/18 Пнд 12:57:47 45453132
>>454527
Что про них прояснять? В них нет ничего такого особенно. Твой пека и то сложнее сделан.

>Каким образом они передают сигналы на землю?
Посредством изучения радиоволн через узконаправленную антенну.

>Что такое вообще наш вай-фай и радио?
Колебание ЭМ-поля, которым задают определенную форму.
Аноним 10/12/18 Пнд 13:06:20 45453333
>>454531
>Колебание ЭМ-поля, которым задают определенную форму.
Волна это. Чего выебываешься? Ученый?) Я тоже ученый, калега)
Аноним 10/12/18 Пнд 14:14:18 45453534
Аноним 10/12/18 Пнд 14:27:59 45453835
15444307989180.jpg (44Кб, 661x505)
661x505
>>454356 (OP)
ЭНИАК – первый типакомпьютер, по факту калькулятор весом 27 тонн. Нахуя он нужен спрашивается? Парочка аутистов со способностями к быстрому счету на страну первого мира всегда найдется, это если так уж необходимо что-то считать (а зачем собственно необходимо так много считать?), либо десяток таких людей как я, с очень хорошей способностью к вычислительным операциям (умножение рандомного целого трехзначного на рандомное четырехзначное за несколько секунд/пару десятков секунд, и за минуту с лишним, если с десятыми и сотыми). Калькулятор-то зачем сдался? ЗАчем так много считать?
Или я чего-то не дочитал про его возможности?
Аноним 10/12/18 Пнд 15:01:35 45454236
>>454538
>умножение рандомного целого трехзначного на рандомное четырехзначное за несколько секунд/пару десятков секунд, и за минуту с лишним, если с десятыми и сотыми
Лол, с такими способностями ты же абсолютно бесполезное говно, тебя можно заменить самым дешёвым китайским калькулятором за 0.02 евроцента, какой суперкомпьютер весом 27 тонн, ты о чём?
Аноним 10/12/18 Пнд 15:09:14 45454337
>>454542
Тогда ты говно! Говно ты! Считать умеет, ха, калькулятор лучше считает! Ты говно ты не нужен!) Тебя заменить можно, подумаешь, умножает, ха, да это я тебе ученый двачер знаток 27 тонн 0,2 евроцента говорю, я знаю, что говорю, я разбираюсь, а ты говно! Вот так просто, заменить тебя. Способности? Ха-ха! Говно, ты о чём? Лол! Лол!!! Смотрите все, ну и говно! Как я его) На место ставить надо таких, талант у них, то же мне, говно они а не талант, вот у меня талант говно на место ставить! У меня есть свое мнение, а ты говно! Заменить! Заменить немедленно!
Аноним 10/12/18 Пнд 15:26:43 45454538
>>454538
Представил себе сборище авторитетных ученых экстра класса по типу Оппенгеймера, а за каждым хвостиком плетется свой личный счетчик-аутист и орет как человек дождя в случае приступа. Но это ничего, всегда есть запасные аутисты в контейнерах, стилизованных под торговый автомат. Кладешь в такой 25 центов и вот тебе – твой ходячий калькулятор..
Аноним 10/12/18 Пнд 16:37:52 45454739
Попадались ли вам пособия по тренировке и работе с образной памятью? Не развитию такой памяти у представителя 95% с нуля, а именно тренировке, расширению её возможностей у нормального человека.
Аноним 10/12/18 Пнд 17:14:20 45454940
>>454544
Ну окей-окей, малец
Как скажешь. Только прежде чем вычислять уроки на завтра доделай
Аноним 11/12/18 Втр 00:38:37 45458441
>>454528
Опять выходишь на связь, долбоеб?
Аноним 11/12/18 Втр 03:41:06 45458942
Поясните за двухпалатный мозг. Бред или не бред?
Аноним 11/12/18 Втр 04:56:35 45459043
>>454584
На связь выходишь? связной дохуя? алло связной радист дохуя? Выходишь? Сейчас зайду так зайду! Ты долбоеб? Ты долбоеб на связь выходишь пидрила русская? Свяжемся? Свяжи меня полностью калега) Радист ты сколько связей ковалетно да? или ионный ты пидарас? а, свинка? опять что ли? долбоеб) какая связь? святой связной радист радистка кэт оторвать тебе шнур гнида? на связь хуй выйдешь потом долбоеб) и опять не выйдешь да ты опять не вышел тебе некуда идти бомжара)
Аноним 11/12/18 Втр 20:10:29 45461744
Нейросети тут?
Аноним 11/12/18 Втр 20:10:47 45461845
Нейросети тут?
Аноним 11/12/18 Втр 20:12:50 45461946
Каким образом в шахматах уравнивается преимущество первого хода?
Аноним 11/12/18 Втр 20:38:22 45462047
Почему анаболитические стеройды не ускоряют рост сухожилия
Аноним 11/12/18 Втр 23:28:31 45462648
>>454619
Первый ход не является преимуществом ни для кого
Аноним 11/12/18 Втр 23:29:23 45462749
>>454620
Не ускоряют у геев а все кочки ну ты понел
Аноним 12/12/18 Срд 00:39:41 45462950
Аноним 12/12/18 Срд 01:40:48 45463351
>>454490
Для существования вселенной нужны создатель или создатели, но речь не о них, а о тебе, и каждом кто ощущает квалиа. Вот атеисты веруют что они всего лишь биороботы, поэтому им нужно строить защиту от нападок на их уютный манямирок. Защита не обязательно должна быть логичной и понятной, ведь у верующего цель не познать реальность, а запутать ближнего своего и себя за одно.
Аноним 12/12/18 Срд 16:49:16 45464852
>>454633
>ощущает квалиа
>атеисты веруют
>приписывает кому-то манямирок, будучи сам давно под горизонтом событий собственного манямирка, горизонт манямирка при этом совпадает с границами черепушки
>употребляет "ближнего своего", при этом c глумливым еблом приватизирует для себя и себе подобных пузанчиков и этику и нравственность, отказывая в их наличии всем прочим
>приписывает кому-то "цель не познать реальность, а запутать ближнего своего", нечаянно забывая, что не далее как в середине 19 века солидные образованные господа с солидными ЧСВ и ПГМ объясняли нахождение больших кусков скал на равнинах вдали от образовавших их горных гряд не оледенением и спуском ледников, а Всемирным Потопом.
Теософ Бомбаст Хвилософ, ты ещё курлыкнуть забыл, цаплю показать и песенку про слоника спеть. Сходи лучше на пруд, положи ласточек в лунку, ведь в твоём упоротом манямирке они должны зимовать на дне водоёмов.
Аноним 12/12/18 Срд 21:33:39 45467853
>>454633
Если ты всерьез такой, а не тралишь, я всегда хотел спросить
> Для существования вселенной нужны создатель или создатели
как ты от рекурсии в этом избавояешься? Пусть для существования нужны создатели. Т.е. получается создатели существуют. Для их существования опять нужны создатели создателей?
Аноним 12/12/18 Срд 22:09:49 45468254
Наукач, почему еще не придумано соединение по типу того же АТФ, которое бы выделяло энергию при добавлении воды распаде? Возможно ли вообще синтез подобного, и если нет, то почему? прогуливаюбиологию
Аноним 12/12/18 Срд 22:15:34 45468355
>>454682
Нитроглицерин подойдет?
Аноним 12/12/18 Срд 22:48:25 45468556
>>454683
Неужели нет аналога, который не взрывался бы от каждого толчка?
или такие вещества и должны быть очень нестабильны? У нас в клетках АТФ тоже взрывается?
Аноним 13/12/18 Чтв 13:15:51 45470957
>>454682
Ты хотя бы сначала изучил процесс фосфорилирования и как энергия передается в биологических процессах, чтоб потом такие вопросы задавать.
Если вкратце, то АТФ просто переносчик энергии. По себе молекула метастабильна, нет она не распадается, просто она находится в локальном минимуме энергии. Для выделения энергии нужен выход из этого локального минимума через реакцию гидролиза. Эта реакция электрохимическая, поскольку АТФ является кислотным остатком, то на нее трудно нацепить еще один электрон, поэтому тут нужен либо градиент потенциала в среде или такие молекулы/ионы, у которых пространственная структура позволяет зафиксировать молекулу АТФ для передачи электрона. Такими веществами являются большинство ферментов и не очень. Собственно именно узкая специализация акцепторов и донор позволяет АТФ быть переносчиком энергии.
Теперь касаемо твоего вопроса. Я же упомянул что, для АТФ для выделение энергии можно использовать внешний потенциал, и если его использовать, то получим аналог обычного электроаккумулятора, да добавок очень хуевого. Вся прелесть АТФ в том, что не запасают энергию, а быстро ее передавать от ферментов к ферментам, а продукты распада использоваться снова для синтеза АТФ.
Аноним 13/12/18 Чтв 14:53:40 45471558
>>454709
Что нужно читать чтобы обладать такими же знаниями?
Аноним 13/12/18 Чтв 15:02:49 45471659
>>454715
Учиться, учиться и ещё раз учиться. (с)
Аноним 13/12/18 Чтв 15:39:38 45472060
>>454715
Ничего особенного, это знания второкурсника медуниверситета. Для таких же охуительных сложных блядь пиздато знаний можешь почитать даже просмотрово Ленинджер биохимия и какой-то был отечественный учебник по молекулярной биологии. Этого хватит. Две книги. И всё.
Аноним 13/12/18 Чтв 16:20:16 45472461
Аноним 13/12/18 Чтв 16:23:51 45472562
13/12/18 Чтв 17:22:25 45472963
>>454720
Мушкамбаров, во!
>>454724
ну это же вроде пучки волокон, это не клетки да и рецепторов у них нет, они и не растут. еще там фиброциты и все такое и ну типа соединительная ткань. рецепторов к стероидам вроде не имеет. хотя хз.
Аноним 13/12/18 Чтв 19:13:48 45473764
По какой причине рис прилипает к кастрюле? Как объясняется данное явление?
Аноним 13/12/18 Чтв 19:38:58 45473965
Аноним 14/12/18 Птн 15:47:20 45477366
Полиэтилен же вроде прозрачный в брикетах? А почему полиэтиленовая пленка полностью размазывает изображение (ну то есть с другой стороны ничего не видно)?
Аноним 14/12/18 Птн 15:58:02 45477467
>>454773
Потому что полиэтилен довольно мягкий и всячески царапается, деформируется и прочая залупа, отчего в нем образуются неоднородности, которые и вызывают рассеивание света.
Аноним 14/12/18 Птн 17:19:14 45477868
- эволюция клетки, от пробионтов к одноклеточным.
- эволюция многоклеточных организмов, от губок к сложным системам.
- эволюция интеллекта, от примитивного интеллекта червя к приматам.
- эволюция социума; среди многоклеточных наибольший успех получают организмы с сложным групповым взаимодействием.
Что дальше? Эволюция групп? Появление роевого интеллекта?
Аноним 14/12/18 Птн 17:23:46 45477969
>>454778
Дальше техноэволюция
Аноним 14/12/18 Птн 17:27:07 45478070
>>454778
Метаэволюция.
Когда целые биоты эволюционирует под заранее уготовленному плану.
Ну или прорыв жизни на неорганику.
Аноним 14/12/18 Птн 17:29:08 45478171
>>454356 (OP)
Прошу объяснить простыми словами, что такое теория относительности.

С примерами про фонари идите нахуй сразу. Что такое слово относительность, что к чему относительно, почему? Для чего? От чего? КАК?!

Аноним 14/12/18 Птн 18:27:23 45478572
>>454781
Короче так, есть классическая теория ньютоновская физика. Хоть там и есть три закона ньютона, но теорию переформулирует в другой виде, а именно через задание совокупности частиц, которые движутся в евклидовом пространстве, на котором определенно некое силовое поле. При этом их движения такие, чтоб удовлетворяли одно строгое условие-принцип наименьшего действие. Попутно евклидово пространство расширяется дополнительным пространством скоростей, и задаем особыми преобразованиями - преобразования систем отчета. Собственно из свойств расширенного пространства и принципа наименьшего действия вытекают ваши законы сохранения и законы ньютона.
Все хорошо, но потом выяснилось что ЭМ поле криво преобразуется при привычных преобразованиях, но сохраняются при других(Преобразования лоренца). Короче теорию начали перепиливать и все выходило в то, что частицы и должны двигаться не 3-х мерном пространстве, а в 4-мерном, да в добавок в этом пространстве по особому меряют расстояния. Ну и пришел Эйнштейн и запили свою физическую теорию, где вместо эвклидового пространства - пространство минковского. Электродинамика теперь приобрела более простой вид, что потом позволило ее еще сильнее обобщить, т.е чтоб эти ваши уравнения максвелла выводились из новой теории, где задано поле.
Это специальная теория относительности, а теперь об общей поговорил.
Жил был математик Риман, занимался он дифференциальной геометрией, и его заинтересовало, что процесс нахождение кратчайших расстояний между точками в произвольном пространстве очень похож на решений уравнений(конкретно это называется нахождение эктрималей лангражиана) движения из физики, и он предложил идею, что любое силовое поле может быть сведено к движению в кривом пространстве.
Ну и на основании этой идеи Эйнштейн начал пилить свою теорию гравитации. Только тут у нас пространство по аналогии с СТО тоже 4-мерное, и с особыми свойствами. Такие пространства называют псевдоориманоскими.

>Что такое слово относительность, что к чему относительно, почему?
Основная идея теория это относительность движения и физических законов, ну или блядь короче во всех ИСО скорость света одинакова. А еще во всех системах отсчета сохраняют некие уравнения движения, тут хорошая аналогия: какую не возьми систему координат, формы графика на ней не изменится.
>Для чего?
Шоб уравнения максвелла во всех ИСО были верны, позже чтоб все поля сводились к одному.
>От чего?
Потому что природа такая, епта
>КАК?!
Выше написал

Аноним 14/12/18 Птн 18:42:34 45478773
>>454785
>в евклидовом пространстве

Дальше не читал. Понял что ты начинаешь нести непонятную хуйню. Попросил же простыми словами. Вот вы такие все одинаковые, те кто понимает что это такое. И когда объясняют, начинают накидывать понятия, чтобы казаться дохуя умными. Ясен пень такое никто за просто так не расскажет. В данном случае денежная единица рассказчика его ЧСВ. Пошел нахуй.
Аноним 14/12/18 Птн 18:45:34 45478874
>>454787
А куда тебе проще? ТО же физическая теория с мат аппаратом, ты думал там будет все понятно на интуитивном уровне?
Аноним 14/12/18 Птн 18:48:49 45479175
>>454788

Не строй дурачка.

Есть всему простое понятие. Ты мне про эвклидовые пространства и пространство миньковского, про какие-то уравнения загнул. Сука в Википедии проще написано, я хоть оттуда почитал про скорость света. И думал, что хоть живое общение поможет мне понять. А в итоге человеческий фактор. Все люди гной ебаный, делают все чтобы потешить свое ЧСВ. Пошел нахуй^2
Аноним 14/12/18 Птн 22:07:11 45479876
>>454781
>что такое теория относительности
Это теория, базирующаяся на таких беспруфных верованях:
1. Якобы есть полностью инерциальные системы отсчета. Они отвечают.
2. Якобы законы природы одинаковы во всех инерциальных системах отсчета. Тут к ним претензий нет, ведь существуют ИСОшки только в маняфантазиях, а значит можно пиздеть про них все что угодно.
3. Якобы существует некая скорость, выше которой низя двигаться. Тут еще можно допустить, но пруфов нет все равно.
4. Якобы эта скорость равна скорости света в вакууме. Они бля будут.
5. Якобы свет обладает волшебными свойствами: всегда летает с этой скоростью относительно всего. Пиздец сказочники.
Есть еще другие верования, но этих достаточно чтобы понять суть теории.
Аноним 15/12/18 Суб 20:40:10 45483577
Почему на яблочном соке в позрачной таре пишут беречь его от воздействия прямых солнечных лучей или даже хранить в тёмном месте ?
Допустим человеку отрубили часть руки или ноги. Оголились поперечные сечения сосудов. В отрубленной части по сути были закругления сосудов. Теперь их нет и по идее кровь должна выливаться с каждым ударом сердца из прямого хода сосуда а в обратный она не попадает. Однако как-то можно остановить кровоток бинтами и потом человек вполне может жить с культой. Это получается в ней выросло новое закругление сосудов, снова соединились прямой и обратный ход ?
Аноним 15/12/18 Суб 21:27:08 45483978
>>454835
>закругления сосудов
Лол, они ж не идут одним своим сечением всё время, а разветвляются, разветвляются и разветвляются, обратно считай уже начинает идти не из сосуда, а из пропитанной кровью губки.
А то что кровь постоянно через них идёт.. ну их заткнут и дальше она идти не будет. А т.к. заткнуто, то и качать её туда не будет.
Аноним 15/12/18 Суб 21:31:30 45484079
>>454356 (OP)
Если отрезать человеку нестоячий хуй, то он сможет после этого драться на адреналине или тут же отрубится от шока или от кровопотери?..
https://www.youtube.com/watch?v=btZoVHmEtUM
Аноним 15/12/18 Суб 22:22:57 45484380
image.png (448Кб, 599x449)
599x449
>>454356 (OP)
Если у всех живых организмов есть предел Хейфлика, вынуждающий клетки делиться лишь ограниченное кол-во раз, то как мы бесконечно, поколение за поколением размножаемся? Ведь для того, чтобы создать дочерний организм, необходимы клетки материнские, на которые, по идее, тоже распространяется этот предел.
Что я понял не так?
Аноним 15/12/18 Суб 23:39:09 45484781
>>454798
>Будто в научных теориях должны быть пруфы
Аноним 16/12/18 Вск 01:07:18 45484982
>>454781
Слово 'относительность' там оттого, что в основе теории лежит 'принцип относительности (Эйнштейна)'. Суть этого принципа в том, что если у нас есть две системы отсчёта, то находясь в одной из них, мы не сможем определить, мы движемся относительно второй или наоборот. Причём подразумевается, что это движение мы должны определить с помощью какого-нибудь эксперимента. Изначально был принцип относительности Галилея в классической механике, который распространяется только на механические явления. Ну то есть ты можешь запускать маятник, кидать мячик в поезде и нихуя не определить, ты движешься относительно дерева, или дерево относительно тебя. Эйнштейн распространил этот принцип на электромагнитные явления (и тут идут математические выкладки, призванные изменить формулы классической механики так, чтобы этот принцип выполнялся).
На самом деле этот принцип лучше было бы назвать 'принципом симметрии физических явлений' или типа того. Относительность там только из-за выше описанного мысленного эксперимента, о котором вспоминают по историческим причинам или для объяснения школьникам. Историческая причина в том, что изначально все это придумал Галилей, когда срался с гелиоцентристами на тему "Земля движется относительно Солнца или наоборот". Поэтому не заморачивайся над одним словом.

Тебе еще может не понравится, что везде пишут про мат модель и не говорят про некий глубинный смысл СТО. Ну вот механика Ньютона для тебя дохуя понятная? Но чем она принципиально отличается от СТО? Вообще-то это тоже мат модель. Ну вот как ты по-простому для себя объясняешь классическую механику? Если ответишь на это вопрос, тебе могут попытаться ьолее-менее понятно для тебя объяснить СТО, а то так нихуя не понятно.
Аноним 16/12/18 Вск 02:14:13 45485083
Слышал как-то байку, что иммунитет человека воспринимает мозг как чужеродное тело, и если бы не гэб, то иммунитет сожрал бы его. И это, мол пруф, что мозг/нервная система был паразитом, который насасывает дофамин, пока тельце потеет. Насколько хуита, и где лучше почитать про эволюцию мозга с современной точки зрения?
Аноним 16/12/18 Вск 10:16:09 45486084
>>454849
>мы не сможем определить, мы движемся относительно второй или наоборот.
Пиздабол он. Мы можем определить, если мерить скорость относительно абсолютной системы отсчета.
Аноним 16/12/18 Вск 14:26:44 45487385
>>454798
>Якобы есть полностью инерциальные системы отсчета.
Про такое слово как абстракция слышал?
>если мерить скорость относительно абсолютной системы отсчета.
Только хохма в том, что такой системы отсчета нету.
Аноним 16/12/18 Вск 14:29:13 45487486
>>454787
>Попросил же простыми словами.
Некоторые вещи не объясняются "простыми словами". Впрочем для того, чтобы понимать СТО достаточно математики 11 класса.
Аноним 16/12/18 Вск 16:10:32 45488587
>>454873
>Только хохма в том, что такой системы отсчета нету.
Вас посетила кефирная инквизиция саентача, на этот раз без штрафа, впредь такой ерунды не говорите.
Аноним 16/12/18 Вск 20:46:07 45490488
1518078505310.gif (3Кб, 483x461)
483x461
У гелия два электрона на одной орбитали (1s²), их волны вероятности совмещены в пространстве. Выходит, электроны могут спонтанно оказаться вплотную и вытолкнуть друг друга из атома?
Заметьте, в учебниках и научпопе как бы невзначай изображают электроны в противоположных точках классической орбиты, чтобы не наводить на мысли о возможности мощного отталкивания.
>Нерешённые проблемы современной физики — Википедия >Неизвестно точное решение уравнения Шрёдингера для многоэлектронных атомов.
Интересно, а приближённое решение как избегает этого парадокса? https://en.wikipedia.org/wiki/Two-electron_atom Это поинтереснее тёмной материи будет, но у "просветителей" я ещё не встречал обсуждения этой проблемы.

Нашёл статью в Википедии:
Теория среднего поля или теория самосогласованного поля — подход к изучению поведения больших и сложных стохастических систем в физике и теории вероятностей через исследование простых моделей. Влияние других индивидуальных компонент на заданный объект аппроксимируется усредненным эффектом, благодаря чему задача многих тел сводится к одночастичной задаче.
Основная идея теории среднего поля — заменить все действия на выбранное тело усредненным или эффективным взаимодействием, которое иногда называют молекулярным полем. Это сводит любую задачу многих тел в эффективной одночастиной задаче. Легкость решения задачи теории среднего поля означает получение определённого знания о поведении системы со сравнительно небольшими затратами.

Такой трюк исключает сближение электронов, объявляя существующим (в виде частицы) только один их них, остальные же мысленно заменяются на усреднённое поле.
Аноним 16/12/18 Вск 21:00:48 45490589
>>454904
Что вообще значит оказаться в одном месте для электронов?
Фигня это всё. Сначала скажи что значит расстояние между электронами, которое вообще есть облако вероятности.
Аноним 16/12/18 Вск 21:55:13 45490690
Аноним 17/12/18 Пнд 00:04:01 45491991
Движение электронов при возникновении напряжения является лишь побочным явлением, а работу(касательно электричества и его использования) совершает что-то другое?
Аноним 17/12/18 Пнд 09:23:20 45493192
>>454905
У тебя (как и у другого анона) передозировка википедией. Измерить координату мы всегда можем (рассеянием фотонами, например), получая одно и только одно из собственных чисел соответствующего оператора. Другое дело, что если ты будешь повторять этот эксперимент с в каком-то смысле идентичными частицами у тебя возникнет распределение вероятности.
Учите матчасть.
Аноним 17/12/18 Пнд 10:10:11 45493393
Почему у спутников планет нет своих спутников?..
Аноним 17/12/18 Пнд 13:50:38 45494594
Аноним 17/12/18 Пнд 14:19:01 45494795
>>454933
По той же причине, почему у Меркурия и Венеры не может быть спутников, система Луна-Земля на гране устойчивости, и орбита вокруг луны не устойчива (большинство посылаемого к Луне говна летали по сложным петлям, или вообще пихали в точку Лагранжа.).
Все дело в приливных силах. Смысл всего этого в том, что у первичного тела тоже есть гравитация, и из-за ее спадании с расстоянием создает приливные силы, делая неустойчивой орбиту вокруг у второго тела, ну еще иногда может деформировать его, заставляя повернутся одной стороной или обращаться вокруг своей оси в кратном соотношении с орбитальным периодом.
Хотя в прицепе если планета будет чуть тяжелее юпитера, а ее спутник будет массой с землю и на дальней орбите, то вполне спутник заимеет свой собственный спутник.
Аноним 17/12/18 Пнд 17:22:03 45495196
>>454906
>https://ru.wikipedia.org/wiki/Реликтовое_излучение#Дипольная_анизотропия
Ты = кусок говна
Реликтовое излучение = океан мочи без внутренних течений в нем
Знание того, что ты-говно не движешься относительно мочи, ничего тебе не даст, а СО, связанная с мочой, ничем от других СО, движущихся относительно нее равномерно и прямолинейно не отличается.
Аноним 17/12/18 Пнд 19:02:36 45496497
>>454951
Ты хочешь сказать что материя вселенной в среднем движется относительно пространства?
Аноним 17/12/18 Пнд 20:21:26 45496998
>>454931
> Измерить координату мы всегда можем (рассеянием фотонами, например), получая одно и только одно из собственных чисел соответствующего оператора. Другое дело, что если ты будешь повторять этот эксперимент с в каком-то смысле идентичными частицами у тебя возникнет распределение вероятности.
Так что такое расстояние между электронами или вообще его координата? Как она определяется, если она каждый раз разная?
Аноним 18/12/18 Втр 01:08:35 45498799
>>454964
Пространства нет, есть только материя и взаимоотношения оной.
Аноним 18/12/18 Втр 02:24:00 454993100
>>454964
Это >>454987 не я отвечал.

Но понятия "движение относительно пространства" в физике вообще не существует.
Аноним 18/12/18 Втр 09:24:15 455001101
>>454987
>Пространства нет
А в чем же тогда содержится материя?

>>454993
Конечно ячейки пространства мы еще не нащупали, чтобы делать привязку системы отсчета к ним, но зато обнаружили реликтовое излучение, которое есть свет великого множества галактик на пределе видимости, что усредняет скорости этих галактик. Если материя во вселенной в среднем покоится относительно пространства, то система отсчета связанная с изотропным реликтовым излучением это практически абсолютная система отсчета.
Аноним 18/12/18 Втр 12:38:50 455006102
>>455001
> А в чем же тогда содержится материя?
В другой материи.

> система отсчета связанная с изотропным реликтовым излучением
Ну погасил ты дипольную компоненту реликтового излучения, перейдя в движущуюся с ~600 км/c. Через некоторое время смотришь - батюшки - опять диполь есть. Что делать будешь? Опять ускоряться?
Аноним 18/12/18 Втр 12:47:53 455007103
>>455006
>Через некоторое время смотришь - батюшки - опять диполь есть.
С чего бы это?
Аноним 18/12/18 Втр 14:47:07 455011104
>>455007
Ты в курсе что у CMB не только дипольная компонента есть?
Аноним 18/12/18 Втр 21:16:47 455041105
Вот видос из сосднего треда про энтропию
https://www.youtube.com/watch?v=tfwSHirk5uo
Я чёт совсем не могу въехать. Они говорят что Энтропия это мера того соклько разных конфигураций молекул дают одинаковое макроскопическое состояние. Для примера смешивают кофе и молоко. Вот по логике. Когда стоп кадр на перемешивании с фрактальными вихрями молока в кофе, это же очень мало конфигураций молекул чтобы была именно такая картинка(состояние), а когда не смешано или полностью смешано там вроде наоборот много разных конфигов молекул дадут такие состояния. Т. е. выходит всё наоборот и я запутался. Теперь вопросы совсем тупые. Чему равна энтропия абсолютного вакуума ? Чему равна энтропия объёма где нет частиц, но есть всякие узлучения. Почему у тепловой смерти вселенной энтропия высокая, если всё застыло. это же порядок ?
>>454835
>Почему на яблочном соке в позрачной таре пишут беречь его от воздействия прямых солнечных лучей или даже хранить в тёмном месте ?
Бамп вопросу.
Аноним 18/12/18 Втр 21:37:21 455043106
>>455041
> Чему равна энтропия абсолютного вакуума ?
Одно возможное состояние - энтропия ноль.
> Чему равна энтропия объёма где нет частиц, но есть всякие узлучения.
Надо считать все возможные каантовые состояния. см. энтропия фотонного газа.
> Почему у тепловой смерти вселенной энтропия высокая, если всё застыло. это же порядок ?
Ну тыж сам написал. Надо считать сколько различных конфигураций дает макросостояние тепловомертвой Вселенной
Аноним 19/12/18 Срд 01:47:18 455051107
Можно ли сделать бионический электронный в полном смысле глаз? То бишь мы монтируем небольшую ебень у человека на лбу, откуда дозировано испускаются рентгеновские лучи / электроны (опционально), и детектор улавливает изменения интенсивности излучения -> выстраивает картинку
Аноним 19/12/18 Срд 02:39:26 455052108
>>455051
Ниче не понял чего ты хочешь. Ты хочешь рентгеновскую лампу / электронную пушку на лоб?
Аноним 19/12/18 Срд 02:47:24 455054109
>>454791
>Есть всему простое понятие.
Давай попробую тебе объяснить, почему объясняя что-то используют абстрактные определения.
Например, пусть я хочу написать, что интеграл от -00 до +00 какого-то выражения равен pi. Такая запись займет всего строчку, но вот если я раскрою все определения и все пределы, что там есть, то выйдет запись на 5 строк из предикатов и логических выражений.
>относительно абсолютной системы отсчета.
Богу дашь фонарик?
Аноним 19/12/18 Срд 02:55:26 455055110
>>454904
>Интересно, а приближённое решение как избегает этого парадокса?
А в чем парадокс? Он возникает ТОЛЬКО в приближенном решении, причем довольно простом. Сейчас попробую объяснить в чем дело, электроны неразличимы, их плотности вероятности не совмещены, у них одна плотность вероятности на всех. А теперь дальше, для двух электронов, волновая функция записывается от шести координат (если спин не брать), волновая функция от N электронов зависит от 3N координат. И как ты правильно заметил, подставив в неё одинаковые координаты для двух электронов, ты получишь 0. Вероятность обнаружить электроны в одной точке пространства 0. Причем в первую очередь на это влияет не сила кулона (она тоже влияет конечно), а тупо принцип запрета Паули.
Аноним 19/12/18 Срд 02:56:19 455056111
>>454919
Поле совершает. Вообще всегда поле совершает всю работу. В любой системе зарядов, энергия это кинетическая энергия частиц + энергия поля.
Аноним 19/12/18 Срд 02:57:34 455057112
>>455001
>реликтовое излучение, которое есть свет великого множества галактик на пределе видимости
Как же я в голосину проорал с этого знатока... Я разбудил кота в три часа ночи.
Аноним 19/12/18 Срд 03:00:55 455058113
>>455041
>Для примера смешивают кофе и молоко. Вот по логике. Когда стоп кадр на перемешивании с фрактальными вихрями молока в кофе, это же очень мало конфигураций молекул чтобы была именно такая картинка(состояние)
Не совсем. Лучше разберись со смешиванием двух газов. Это проще.
Аноним 19/12/18 Срд 05:24:56 455060114
Аноним 19/12/18 Срд 10:26:25 455065115
Блятб, ладно в среде свет распространяется прямолинейно, т.к. она по большей части пуста, ок. Но почему угол падения равен углу отражения, и луч после отражения остаётся лучом??
Аноним 19/12/18 Срд 10:28:22 455066116
>>455056
Т.е. движения электронов лишь побочное и вредное явление от которого нужнои как-то можно? кроме очевидного увеличения напряжения избавляться?
Аноним 19/12/18 Срд 13:55:55 455070117
>>455066
А схуяли по-твоему в ЛЭП такие напряжения?
Аноним 19/12/18 Срд 14:01:29 455071118
>>455043
>Одно возможное состояние - энтропия ноль.
Точно вот это ? Точно определено само поняте энтропия для абсолютного вакуума ?
>Надо считать сколько различных конфигураций дает макросостояние тепловомертвой Вселенной
Воображаемая трёхмерная вселенная где уже наступила тепловая смерть. Надо полагать там осталось какое-то вещество ? Мы как то посмотрели как вещество там распределено и построили конкретную скалярную функцию F(x,y,z), она задаёт массу в этой вселенной(процессы остановились, излучений допустим нет, больше не знаю какие макропараметры нужны; типо всё в этой вселенной из одного вещества). Энтропия такой вселеной это конфигурации частиц дающие макропараметр масса как в функции F или все конфигурации частиц дающие скалярные функции массы соответствующие тепловой смерти данной вселенной ?
Аноним 19/12/18 Срд 14:06:14 455072119
ReflRefr.gif (222Кб, 350x350)
350x350
Аноним 19/12/18 Срд 14:10:03 455073120
Аноним 19/12/18 Срд 14:12:30 455074121
>>455071
> Точно вот это ? Точно определено само поняте энтропия для абсолютного вакуума ?
Ну да.
> F(x,y,z)
Ты путаешь микро- и макро- описания. F(x,y,z) -- это по-сути микроописание -- ты так задаешь положения всех частиц. Энтропия -- это количество микросостояний (логарифм) в заданом тобой макросостоянии. Если у тебя нет макроописания в котором много микроописаний, то энтропия просто не определена.
Аноним 19/12/18 Срд 14:12:33 455075122
>>455073
А, так по этому принципу сверхпроводники действуют, выдавливают из себя поле наружу, и никакие электроны никуда нетдвигаются паразитно.
>>455072
Охуенно, спасибо ананочик

Ох ебать сегодня день открытий.
Аноним 19/12/18 Срд 14:13:10 455076123
>>455073
И чего ты хочешь? Похвалы? Ну маладец!
Аноним 19/12/18 Срд 14:16:41 455077124
>>455076
Там вопрос. Похволы за вопрос? Совсем йобобо?
Аноним 19/12/18 Срд 14:34:48 455078125
Инженерия и дизайн это два подхода к одному и тому же или совсемразные вещи ? Оба являются направлениями творческой деятельности человека по созданию чего-либо, но соответствие результата поставленной цели в инженерном деле определяется строгим формальным критерием(подходят ли физические свойства объекта под заданные условия), а в дизайне - субъектвиным критерием(достаточно красиво/эстетично). А искусство оличается от дизайна тем, что в нём вобщем-то и цели нет ?
Аноним 19/12/18 Срд 15:16:15 455079126
>>455072
Анончик, а откуда берётся прямолинейность распространение света от источника, если фотон после излучения существует одновременно везде в увиличивающейся каждую секунду на 300т.км., и появлчется только в момент столкновения с чём-то?(это вообще так?)
Аноним 19/12/18 Срд 15:20:17 455080127
>>455072
Т.е. эти толстые линии это не количество света(ну как следствие и оно), а первоначально-области наибольший вероятности?
Аноним 19/12/18 Срд 15:45:44 455081128
Чтобы МАТАНом овладеть нужна ли хорошая память или только способность врубаться?
Аноним 19/12/18 Срд 16:12:02 455083129
>>455079
> прямолинейность распространение света
Гугли пятно Пуассона и охуевай
Аноним 19/12/18 Срд 16:15:49 455084130
>>455081
Способность врубаться это тоже в каком-то смысле память. Я отдельные факты очень плохо запоминаю. А вот когда они по-ассоциации в систему выстраиваются, то другое дело
Аноним 19/12/18 Срд 17:14:04 455085131
>>455078
А про инженерный дизайн ты не слышал значт?
Аноним 19/12/18 Срд 18:09:42 455088132
>>455052
Именно. Только маломощную
Аноним 19/12/18 Срд 18:14:59 455089133
>>455088
Ну вытащи из старого телевизора и примотай изолентой
Аноним 19/12/18 Срд 19:00:53 455095134
Сай, вы могли бы самостоятельно составить формулу для нахождения площади/объема/периметра какого-либо неизвестного вам типа фигур? (Представьте, что не знаете, как находить площадь того же тороида) 4п^2rR Я говорю не о банальном
"смоделируй и сравни 50 фигур, ищи отличия и закономерности", а об интуитивном понимании математических формул и знаков.
19/12/18 Срд 19:04:16 455096135
>>455095
какую хуйню несет долблоеб
Аноним 19/12/18 Срд 19:06:01 455097136
Что такое теория относительности?

Поспорил с одним человеком, читаю вики ничего не понимаю. Мб кто разжует без этих ваших заумных понятий.
Аноним 19/12/18 Срд 19:08:55 455098137
>>455095
Ты про поверхностные интегралы не знаешь, да?
Аноним 19/12/18 Срд 19:09:39 455099138
>>455098

А при чем тут это?
Аноним 19/12/18 Срд 19:09:39 455100139
>>455097
О чем поспорил то
Аноним 19/12/18 Срд 19:10:21 455101140
>>455100
На зачет. Очевидно же.
Аноним 19/12/18 Срд 19:10:33 455102141
>>455099
> формула для нахождения площади/объема/периметра
... ЛЮБОГО типа фигур
Аноним 19/12/18 Срд 19:11:31 455103142
>>455101
Я спросил не "на что" а "о чем"
Аноним 19/12/18 Срд 19:13:10 455104143
>>455103

Ты тупой? Я же написал: Теория относительности.
Аноним 19/12/18 Срд 19:17:55 455105144
>>455104
Понял. Объясняю.
Теория относительности это такая теория, которая относительности.
Аноним 19/12/18 Срд 19:23:04 455106145
>>455105
Спасибо.

пошел нахуй
Аноним 19/12/18 Срд 21:00:48 455108146
>>455089
Типо как в Киберпуке 2077? мне б миниатюрную...
Аноним 19/12/18 Срд 22:52:02 455112147
Метр значит это выведенная из расстояния от Барселоны до Дюнкирхена единица длинны, а потом так через овердохуя лет при измерении скорости света она ВДРУГ оказывается таким красивеньким практически идеально кругленьким числом? Хммм...
Аноним 19/12/18 Срд 22:54:28 455113148
>>455112
> красивеньким практически идеально кругленьким числом
299792458 m/s
Поясните за физику Аноним 19/12/18 Срд 23:16:17 455114149
грузовик.jpg (83Кб, 799x600)
799x600
Сап /sci/, такой тупой вопрос:
Предположим я йух, который не осилил правила дорожного движения и переходит дорогу в неположенном месте. В меня въёбывается грузовик. Если мне известна скорость грузовика в момент удара, его масса и соответственно моя масса, то можно ли как-то по импульсу (или вообще как-то) определить, какой путь я пролечу, прежде чем сдохну?
Аноним 19/12/18 Срд 23:37:36 455115150
>>455112
>она ВДРУГ оказывается таким красивеньким практически идеально кругленьким числом? Хммм...
Да, как пи=3
Аноним 19/12/18 Срд 23:41:20 455116151
>>455113
Да, слишком кругленькое для случайности.
Аноним 20/12/18 Чтв 00:09:35 455118152
Почему, если металлический куб (да даже просто сетка) полностью экранирует ЭМ волны, невозможно экранировать гамма-излучение, которое то же самое ЭМ излучение обычной металлической шкатулкой?
Аноним 20/12/18 Чтв 00:52:00 455119153
>>455118
>Почему, если металлический куб (да даже просто сетка) полностью экранирует ЭМ волны
Не экранирует.
Аноним 20/12/18 Чтв 00:58:51 455121154
>>454356 (OP)
Вот зарегистрировали уже десяток слияний черных дыр (грав. волны). Вопрос - разве рядом с черными дырами время не идет столь медленно, что события самого слияния (а значит и сигнала в виде грав. волн) удаленному наблюдателю надо ждать бесконечно долго?
Аноним 20/12/18 Чтв 01:01:40 455123155
>>454843
>Предел Хейфлика - граница количества делений соматических клеток
>соматические клетки - клетки, составляющие тело (сому) многоклеточных организмов и не принимающие участия в половом размножении.
Аноним 20/12/18 Чтв 08:02:40 455130156
>>455121
Нет, ведь гравитационная волна возникает не на горизонте событий, а между дырами
Аноним 20/12/18 Чтв 08:02:48 455131157
Если свет не имеет массы, то почему на него действует гравитация?
Аноним 20/12/18 Чтв 08:05:31 455132158
>>455131
Потому что гравитация искажает и пространство, через которое летит свет
Аноним 20/12/18 Чтв 10:58:36 455140159
1545292455381-9[...].jpg (38Кб, 1024x1003)
1024x1003
Почему элеетрон не падает на ядро. Пиздец, пытался загуглить, а дают какую-то непонятную хуйню про пртнцип неопределенности, Шредингера и еще какую-то хуету.
Какого хуя эта квантовая физика такая непонятная? Даже теория относительности более понятная, чем эта хуйня!

Обьясните плес, почему электрон не падает на ядро и почему хуета с пика аннигилируется. А еще, как получают этот позитрон и какого хуя происходит аннигиляция. Аннигиляция будет происходить между протоном и электроном?
Аноним 20/12/18 Чтв 11:47:49 455141160
15451656505180.png (274Кб, 540x698)
540x698
>>455119
>Не экранирует.
А если из сверхпроводника первого/второго рода?
А если из металлического водорода?
А если проэкранирую?
Аноним 20/12/18 Чтв 12:25:06 455143161
Buckminsterfull[...].gif (516Кб, 300x300)
300x300
Fullerenec540.png (65Кб, 665x725)
665x725
15449933120521.jpg (92Кб, 1024x682)
1024x682
Как-то у вас этот перекат совсем скучным и унылым получился. Разбавлю немного.

Дана система чёрных дыр, примерно как на пикрилах. Эти ЧД аккуратно (но совсем не обязательно медленно) сближают так, чтобы расстояния между соседними ЧД в ячейках одновременно оказались примерно равными границе Шварцшильда для каждой из пар ЧД. Внимание, вопросы:

1) Что произойдёт с внутренней областью? Станет ли она изолированной относительно внешней части Вселенной, или можно будет лицезреть выворачивание сферы наизнанку? Наводящий подвопрос: а у пустого пространства внутри этого чёрнодырового фуллерена вообще есть причина "рассасываться" ЧД?

2) Что произойдёт с гравитационными волнами? Верно ли, что часть их "застрянет" во внутренней области пространства? Как это может повлиять на внешние проявления новообразовавшейся ЧД?

3) Если запутать частицы, и запустить одну из них в центр пустоты будущей ЧД (а другую оставить при себе), сможем ли мы наблюдать "пугающее дальнодействие" в случае отрезанной (изолированной) системы?

4) Если внутренняя область не исчезнет (я этого не утверждаю и не отрицаю здесь), верно ли, что испарение ЧД будет происходить вдвое быстрее нормы?
Аноним 20/12/18 Чтв 12:41:11 455145162
15450771583520.jpg (102Кб, 604x561)
604x561
>>455143
>между соседними ЧД в ячейках одновременно оказались примерно равными границе Шварцшильда для каждой из пар ЧД
Ой.
для каждой из СМЕЖНЫХ ПО ГРАНИ пар ЧД
Мои извинения.
Аноним 20/12/18 Чтв 12:44:38 455146163
>>455145
Или, кстати, даже по ребру, для случая ячеек с количеством сторон n>6.
Аноним 20/12/18 Чтв 12:48:26 455147164
>>455143
А они не ебнутся друг на друга под действием гравитации?
мимо дебил
Аноним 20/12/18 Чтв 13:06:05 455148165
>>455140
>Почему элеетрон не падает на ядро.
Ну как можно упасть, если у тебя траектории нет?
Аноним 20/12/18 Чтв 13:11:37 455150166
15356364020860.png (2627Кб, 828x1217)
828x1217
>>455147
>А они не ебнутся друг на друга под действием гравитации?
Обязательно ёбнутся.
И сольются. Попарно.
Т.к. для каждой из пар по ребру будет момент, что их суммарная масса делённая на расстояние между ними станет меньше гравитационного радиуса.
Вопрос именно в том, что за нах по итогу произойдёт. Разрежет ли пространство на две изолированные области этим покрывалом из ЧД или нет.
Аноним 20/12/18 Чтв 13:31:02 455152167
>>455143
На сколько я помню, нет пока точного описания внутренностей чд, значит мы не знаем, что там внутри твоего чернодырного фуллерена будет
поправьте, если ошибаюсь
Аноним 20/12/18 Чтв 13:57:09 455154168
>>455150
Бля, но ведь они находятся на одинаковом расстоянии друг от друга. Каждую ЧД с одинаковой силой тяну в разные стороны три других чд. Тогда они останутся на месте? Или сам шар будет сжиматься?
дебил
Аноним 20/12/18 Чтв 13:59:05 455155169
>>455148
Почему траектории нет? Например, я ебну по атому, задену его электронное облако и электрон улетит нахуй или упадет на ядро. Почему такого не произойдет?
Аноним 20/12/18 Чтв 14:05:50 455156170
>>455155
>Почему траектории нет?
Она принципиально неизмерима.
Аноним 20/12/18 Чтв 14:17:43 455157171
>>455155
Облако есть. А траектории нет. Волновая функция электрона спокойно себе вокруг ядра сидит.
Аноним 20/12/18 Чтв 15:12:31 455158172
>>455155
Из определения, траектория это некий 1-мерный объект.
Но электрон в атоме нельзя описать через 1-мерное движение. В фейманском формализме он одновременно движется по всем мысленным траекториям, т.е его движение нахуй бесконечномерное. Впрочем если мы будем на него часто "смотреть", он будет двигаться на некой средней. В атоме же электрон находится в потенциальной яме и ему не хватает средней энергией, чтоб выбраться от тудова. В некотором смысле в атоме он упал и лежит на самом дне и ниже ему некуда падать. Если мы захотим часто "посмотреть" на него, то "среднее" размазывается по некой области, называемой орбиталью или электронным облаком. На самом деле орбиталь должна учитывать одновременно еще и пространства импульсов, но на это забивают хуй.
Короче так, не всякое возмущение может изменить состояние электрона. Это довольно важное важное свойство, из-за проистекают очень много свойств вещества как прозрачность и рассеивание света. К примеру только фотон нужный длинны волны и направление импульса может расшатать систему так, чтобы оторвать электрон или посадить его на другую орбиталь, в противном случае атом примет и моментально переизлучит его в том же направлении.
Аноним 20/12/18 Чтв 15:23:44 455161173
А учёные думают что определённое количество света в определённом объёме превратится в чёрную дыру только потому что современные формулы и модели не могут описать бОльшую плотность энергии?
Аноним 20/12/18 Чтв 15:26:11 455163174
>>455161
Ну как бы... в уравнение Эйнштейна входит тензор энергии-импульса, который как бы и отвечает за гравитацию. А электромагнитное поле в этом плане ничем не примечательное.
Аноним 20/12/18 Чтв 15:40:11 455165175
>>455161
Короче так, в ОТО гравитацию задает не масса, а соотношение импульса-энергии. Причем весьма хитрым образом. Нам надо взять некую поверхность в (3,1) пространстве и по всему объему посчитать энергию-импульс, и уже взятый интеграл по границе поверхности будет наша родимая гравитация. Однако из-за того, что масса инварианта, мы получаем, что в стационарных граничных условиях энергии-импульс зависят в основном от массы. и следовательно гравитацию определяет распределение массы. Впрочем ОТО допускает, что мы можем создать такой световой поток, что можно создать ЧД епта, но это решение не стационарно, т.е полученная ЧД непостоянна по времени, - мы сможем наблюдать ее только мгновенье.
Аноним 20/12/18 Чтв 15:53:35 455168176
>>455165
>мы сможем наблюдать ее только мгновенье.
Т.е. она будет неустойчива, и будет существовать только в то мгновение, когда всё необходимое количество света окажется в одном необходимом объёме, а потом он разлетится дальше по своим делам?
Аноним 20/12/18 Чтв 16:04:33 455170177
>>455168
Таки да. Более того, чтоб время жизни было продолжительное нужен не только некий поток излучения, а его постоянно возрастание. Т.е нужно свет постоянно усиливать со временем.
Аноним 20/12/18 Чтв 16:10:00 455171178
>>455170
1)какая тогда это вообще чёрная дыра.
2)вроде по определению чёрная дыра это область пространства, искривлённая достаточно, чтобы из неё не мог выйти свет, как он тогда может разлететья?
Аноним 20/12/18 Чтв 16:27:34 455173179
>>455171
Не люблю я определение через невыход света, скорее это область протестантства-времени в котором времени-подобный интервал становится пространственно-подобным. Из этого утверждения гораздо больше пользы выходит, но я отвлекся.


По идеи это должно выглядит так: пускаем наш йоба-фонарь, который все лучи направляет в одну точку, которую попутно просвечиваем мимо-лазером. После внезапной вспышки, лазер позволяет зарегистрировать, что он не смог измерить точку.
Или для стороннего наблюдателя. Йоба вспышка, а после него оскалился горизонт событий и мигом исчез. Время наблюдения ЧД довольно маленькое, меньше чем порождающий свет успеет пройти за горизонт.

Однако тут есть одна проблема, которая ссылается на такую вещь как уничтожение информации в ЧД, в чистой ОТО часть энергии света должна будет неизбежно проебаться. Как это разрешать, зависит от квантовой гравитации.
Аноним 20/12/18 Чтв 17:13:51 455175180
>>455171
Подкину тебе другое размышление (ответ на который кстати есть):
Массивная сфера, которая имеет радиус чуть больше радиуса Шваршильда (тоесть почти черная дыра). А так же две инерциальные системы, в одной она покоится, а другая система движется с такой скоростью, что в ней кинетической энергии сферы достаточно, чтобы она стала черной дырой. Один наблюдатель видит объект, а для второго этот объект черная дыра.
Аноним 20/12/18 Чтв 17:57:46 455176181
>>455175
Какая нахуй инерциальная система в ОТО?
Поток импульса через поверхность только "ослабевает" гравитационные силы. Тем более в уравнение только учитывается плотность полная энергии, и в добавок для симметрии тензора добавляют поле киллинга, которую кинетическую энергию в ноль уводит. Так что какую кинетическую энергии не сообщи телу, это превратит ее в ЧД.
Аноним 20/12/18 Чтв 18:23:31 455177182
>>455176
Околосветовой объект не будет черной дырой. Но выглядеть он будет как черная дыра.
Аноним 20/12/18 Чтв 19:17:18 455180183
>>455140
> Аннигиляция будет происходить между протоном и электроном?
Не будет. Они не являются по отношению друг к другу античастицами.
Аноним 20/12/18 Чтв 21:15:08 455188184
Что будет, если у человека внезапно гены во всех клетках поменяются на гены другого человека?
Какие недостатки и плюсы у человеческого скелета 20/12/18 Чтв 21:21:31 455189185
Какие недостатки и плюсы у человеческого скелета?????
Аноним 20/12/18 Чтв 21:57:10 455190186
>>454931
cука, не знаешь что ответить
хотя сам говоришь "Учи матчасть"
иронично
Аноним 20/12/18 Чтв 21:58:57 455191187
>>455060
а ты чего такой не добрый?
Аноним 20/12/18 Чтв 22:03:44 455192188
>>455189
Плюсы - прочен, при этом очень лёгок. Куча рычагов обеспечивает скорость или силу там, где это нужнее, огромное колличество разных движений
Минусы - ультимативной защиты не даёт, некоторые места излишне сложны, много уязвимостей, изнашивается к 30-40 годам так как обычно столько не жили
Аноним 20/12/18 Чтв 22:05:07 455193189
>>455188
Если у того не было всяких генетически обусловленных заболеваний, то ничего не изменится сразу. Через некоторое время поменяется оттенок и структура кожи, волосы, ногти.
Аноним 20/12/18 Чтв 22:29:36 455195190
почему сверхпроводоники сверхпроводят?
я конечно знаю, что электроны объединяются в пары и образуют частицы у которых теперь спин не дробный, а целый. А частицы с целым спином образуют конденсат, то есть все оказываются в одном и том же квантовом состоянии. Получается такая сверхтекучая электронная жидкость.
но стоит спросить - почему бы им объединяться?
Аноним 20/12/18 Чтв 22:55:00 455196191
>>455195
>я конечно знаю, что электроны объединяются в пары и образуют частицы у которых теперь спин не дробный, а целый. А частицы с целым спином образуют конденсат, то есть все оказываются в одном и том же квантовом состоянии. Получается такая сверхтекучая электронная жидкость.
Стоп, а разве не потому что электрическое поле выдавливается из сверхпроводника и течёт снаружи его, где нет сопротивления?
Аноним 20/12/18 Чтв 23:02:10 455198192
>>455196
Это просто параллельные описания.
Аноним 20/12/18 Чтв 23:02:35 455199193
Аноним 21/12/18 Птн 06:31:18 455200194
>>455188
В зависимости какие гены поменяются. В общем ничего особого не пройдет, будет медленно меняться биохимия.
Впрочем у человека есть свой уникальный набор антител, которые живут свой жизнью и на внезапные изменения начнут реагировать, в итоге есть вероятность развития аутоиммунного заболевания, поскольку гены, кодирующие мембранные белки, у каждого человека уникальны, а антитела дрессируются узнавание их как друзей.
Аноним 21/12/18 Птн 11:37:50 455204195
Аноним 21/12/18 Птн 13:06:33 455208196
>>455204
Ты свою випикедию сам-то почитай...
Аноним 21/12/18 Птн 13:17:42 455209197
F433121F-7364-4[...].jpeg (21Кб, 400x400)
400x400
можно ли пожарить еду на биокамине?
Аноним 21/12/18 Птн 15:30:28 455210198
>>455209
Можно, это ж не галограма.
Аноним 21/12/18 Птн 15:33:47 455211199
24204350.jpg (76Кб, 660x483)
660x483
Поясните за птиц. Когда они появились? От кого произошли? Почему некоторые источники указывают, что настоящие птицы появились только в конце мела; но в других сказано, что птицы уже существовали в середине триаса, когда млекопитающих-то полноценных ещё не было? Если птицы и динозавры - параллельные ветви эволюции, то почему многие динозавры были в перьях и к мелу обрели птицеподобные черты? А если птицы и произошли от динозавров, то почему первые настоящие предки птиц - археоптериксы и им подобные - возникли гораздо позже, чем предполагаемые птицы триаса? Птицы в процессе эволюции возникали несколько раз или как?
Аноним 21/12/18 Птн 15:51:58 455212200
>>455189
Плюс: есть кифозы-лордозы, то есть амортизация, ноги длинные и на них можно стоять.
Минусы: рожать сложнее, чем животным, хрупкий зуб около головы, есть много болезней типа радикулита.
Аноним 21/12/18 Птн 15:54:24 455214201
Тупой вопрос, но все же, почему темная материя не может быть антиматерией?
Аноним 21/12/18 Птн 16:00:07 455216202
>>455214
Потому что заряжена
Аноним 21/12/18 Птн 16:06:41 455217203
image068.png (39Кб, 300x232)
300x232
Почему внутренняя индукция не выходит на насыщение в петле Гистерезиса?
Аноним 21/12/18 Птн 16:23:28 455218204
>>455214
Потому что свойства антиматерии такие же как у обычной материи, у темной материи вполне конкретные свойства - они ТЯЖЕЛЫЕ, ЭЛЕКТРОМАГНИТНО НЕ ВЗАИМОДЕЙСТВУЮТ И ОНИ "ХОЛОДНЫЕ".
Аноним 21/12/18 Птн 16:27:53 455219205
>>455217
Как это это не входит? Она же задает ширину площадь петли, епта.
Аноним 21/12/18 Птн 16:38:31 455220206
>>455216
Наверное, ты имел в виду не заряжена.
Аноним 22/12/18 Суб 01:04:55 455244207
>>454378
Потому что ты сможешь услыщать его слова, и убить его, до того, как он их скажет. Парадокс.
Аноним 22/12/18 Суб 01:05:36 455245208
>>454371
Как силы трения нет? Сила трения качения, довнич.
Аноним 22/12/18 Суб 01:35:41 455246209
15450334213690.jpg (34Кб, 604x525)
604x525
15449575683510.png (1260Кб, 900x746)
900x746
>>455152
> нет пока точного описания внутренностей чд, значит мы не знаем, что там внутри твоего чернодырного фуллерена будет
А разве я спрашиваю о внутренности ЧД? Я спрашиваю об области пространства, вдруг внезапно ограниченного чёрнодырными "плоскостями". Не одно и то же. #этодругое


>Каждую ЧД с одинаковой силой тяну в разные стороны три других чд. Тогда они останутся на месте? Или сам шар будет сжиматься?
Нет, им изначально придаётся импульс, необходимый для преодоления взаимных пространственных ятвойтрубошатаний. Можно считать, что шар квазиравномерно сжимается. Ну, пусть для примера, каждая из чёрных дыр на реактивной тяге своего сверхмощного джета.
Аноним 22/12/18 Суб 08:17:02 455248210
>>455244
Ты тупой и или траллишь?
Аноним 22/12/18 Суб 09:15:32 455249211
>>455219
оно же дальше прямолинейно еборить должно, не?
Аноним 22/12/18 Суб 11:17:23 455250212
изображение.png (391Кб, 1300x1390)
1300x1390
Если сожрать небольшой листок бумаги вроде этого цельным куском, то он целиком переварится? И как быстро?
Аноним 22/12/18 Суб 14:24:25 455255213
moo1.png (1700Кб, 1920x1063)
1920x1063
Взгляните на глаза пикрил-чебурашки. Такие глаза, со зрачками размером со всю глазную щель, охватывающими полем зрения всё, что находится перед ними, возможны в природе? Полнее раскрою вопрос: то бишь зрение не точечное, как у нас, а цельная картинка,
возможна?
Аноним 22/12/18 Суб 14:51:49 455257214
>>455255
>размером со всю глазную щель, охватывающими полем зрения всё, что находится перед ними
Я наверное открою секрет. Но ты тоже охватываешь всё перед собой, ведь у тебя линза в глазу есть.

>зрение не точечное, как у нас, а цельная картинка,
>возможна?
Чисто с точки зрения оптики, ты не сможешь разобраться откуда свет пришел. У тебя тогда куча лучей будет прилетать из каждой освещенной точки.
Аноним 22/12/18 Суб 17:34:47 455261215
Аноним 22/12/18 Суб 18:07:30 455263216
>>455261
"Master of orion 2016" красные злые пидарасы а-ля гроксы – автолевелинговые, в хлам разъебали моих уберчитерных псилонов со 100500 лучшими кораблями.
Аноним 22/12/18 Суб 22:11:13 455273217
>>455255
нет. большие глаза занимают много места в голове, а значит много места у мозгов. в добавок огромная голова при малом тельце. сложность питания мозга растёт по экспоненте по мере его роста, ибо в глубь всё сложнее дотянутся, а как такое малое тельце будет его кормить не ясно.

Ну если только искусственно такую жизнь создать, хотя такие глаза и особенно третий не понятно зачем
Аноним 22/12/18 Суб 22:41:41 455274218
>>455273
>искусственно создать
А ведь это в самом деле так в данном случае. По лору игры псилоны – терпилы, над которыми проводили опыты. Что забавно, мозги у них не от опытов таковыми стали. Изначально иx тела были крепкими и два метра в высоту. Исходя из того, что с ними способны соперничать те же люди, обладающие малым мозгом – мозги синеньkих гуманоидов неэффективны.
Утром меня заинтересовала конкретно такая мысль: "обязательнo ли для оперативной обработки очень широкого поля зрения (не такой узкой "точки" как у нас, а охвата всего, что перед глазами махом) наличие дюжих интеллектуальных способностей, годных мозгов, что в состоянии быстро оценивать множество деталей" От этой мысли почувствовал неполноценность себя как человека, получается, что люди такие тупые, что не обладают присущим интеллектуальным существам типом зрения.
Аноним 22/12/18 Суб 22:53:38 455275219
>>455274
Тут ещё проблема в таком размере головы на такое малое тельце в том, что не ясно как их шея может её держать.

А для оперативной обработки широкого поля зрения опять же мозг нужен. Ибо самим глазам ничего обрабатывать не нужно. Они получают сигнал и тут же его отправляют. У нас нюх хуже чем у собаки, просто потому, что у собаки та часть мозга, что отвечает за обоняние занимает куда больше места. А мы в итоге пожертвовали этими частями дабы дать пространство для более разумных частей мозга.

По всей видимости, со зрением должна быть та же история. И опять же, сложность системы будет расти просто на порядки по мере более обработки информации со всё более широкого поля зрения.
Аноним 25/12/18 Втр 01:47:13 455436220
Возник тупой вопрос: когда что-то отрывается от гравитирующего тела и тратит энергию, эта затраченная энергия оказывает влияние на само тело?
Аноним 25/12/18 Втр 03:41:49 455439221
Объясните в кратце, откуда столько измерений в теории струн? Где они существуют, и существуют ли вообще? Там есть жизнь? Как это вообще понять?
Аноним 25/12/18 Втр 05:04:33 455440222
>>455439
просто математические костыли.
Аноним 25/12/18 Втр 05:06:25 455441223
>>455436
как отрывается? взлетает на ракете? осколок откалывается? Ну оказывает конечно. Только в случае полётов в космос околонулевое, если же луна отколется и улетит, то покалбасит как следует.
Аноним 25/12/18 Втр 07:07:12 455443224
>>455439
А тебя не смущает, что в обычных теориях три измерения? Откуда они?
Аноним 25/12/18 Втр 08:04:06 455444225
1545702073919.jpg (346Кб, 1072x799)
1072x799
>>454356 (OP)
Это правда, что любовь быстрее света?
Аноним 25/12/18 Втр 10:14:49 455455226
>>455444
Нет. Любовь движется со скоростью света.
Аноним 25/12/18 Втр 10:31:11 455456227
>>455439
11 на 11 (ну еще дохуя матриц вида 11x11x11x11x...x11, но они все общего вида и всегда одинаковые) это размер матрицы, которая определена в каждой точке бесконечномерного пространства.
Аноним 25/12/18 Втр 10:55:05 455457228
>>455444
Нет, ты сначала должен как-то узнать об объекте любви
Аноним 25/12/18 Втр 11:52:09 455463229
Так, анон, пока ты не съебал по причине наплыва ботов, такой вопрос: курение сигарет замедляет рост подростка? Если да, это смолы и пропитка мочой делают, или никотин?
Аноним 25/12/18 Втр 11:59:09 455465230
>>455463
Нет, можешь не курить и быть карланом.
Аноним 25/12/18 Втр 12:38:20 455467231
>>455465
Да у меня родственница шкура тупая курит, вот и думаю. Гены у неё нормальные судя по двум братьям 185+, то бишь ей в них прописан рост 174+, но чую -20см получит как следствие курения.
Аноним 25/12/18 Втр 12:42:09 455468232
>>455444
У меня аутиста разъёбаны рецепторы окситоцина, не могу понять твои обезьяньи чувства.
Аноним 25/12/18 Втр 12:43:26 455469233
>>455467
Тут куда вероятнее, что мамка изменила папке с соседом карланом, чем из-за курения
Аноним 25/12/18 Втр 22:51:22 455510234
15455784258880.png (241Кб, 500x411)
500x411
15455962400490.png (793Кб, 1280x635)
1280x635
>>455444
>Это правда, что любовь быстрее света?
Это возможно лишь для случая падения Любви в Чёрную Дыру.
Аноним 25/12/18 Втр 22:59:31 455511235
15454257695590.jpg (103Кб, 540x960)
540x960
>>455463
Витамин PP. Гугли сам, по памяти (перепроверь!) скажу, что вроде бы регулярное применение извне атрофирует эндогенную его выработку в более щадящей и приемлемой форме, которая ответственна за рост в том числе. Т.е. при прочих равных остаёшься мелкой навальной кучкой школьника, да.
Аноним 26/12/18 Срд 00:15:15 455517236
image.png (746Кб, 600x848)
600x848
>>454356 (OP)
Каким именно образом сода влияет на фармакокинетику никотина в снюсе? Как именно катионы натрия и анионы гидрокарбоната влияют на никотин/мембраны клеток, что скорость всасывания увеличивается?
Аноним 26/12/18 Срд 00:31:40 455519237
Что будет с человеком, если каким-нибудь образом вызывать у него боль, без физических последствий. Типа как заклятие круциатуса из гп, или кластерных головных болей, только постоянно и без возможности убить себя.
Аноним 26/12/18 Срд 00:36:22 455520238
>>455519
Болевой шок, либо диссоциотивное расстройство с реактивным течением (части мозга/психики будут хуже друг с другом связываться), либо потеря сознания.
Аноним 26/12/18 Срд 00:42:38 455521239
>>455520
Я читал, что болевого шока не бывает, типа это такое обывательское название травматического шока. По примеру АК-47 и АК-74
Аноним 26/12/18 Срд 00:46:36 455522240
>>455521
А я читал что на заборе пишут "Хуй" Боль как ни крути - физиологический процесс завязанный на ноцицептине, норадреналине, глутамате и хуй знает чём ещё. При гиперстимуляции нейронов возникнет ответная реакция на торможение, т.к кол-во вторичных посредников не бесконечно. Там либо шок, либо смерть от инфаркта/инсульта в конечном итоге.
Аноним 26/12/18 Срд 00:48:53 455523241
image.png (186Кб, 477x268)
477x268
>>455520
>Что будет с человеком, если каким-нибудь образом вызывать у него боль, без физических последствий
>Болевой шок
Домохозяйка, что ты забыла в наукаче?

>>455519
Правильный ответ - цепи, которые будут постоянно воспринимать боль, атрофируются, и ты перестанешь её воспринимать. Гораздо интереснее будет если боль будет приходить и уходить. Тогда у тебя просто будет очень плохой характер.
Аноним 26/12/18 Срд 00:50:09 455524242
>>455522
>Там либо шок, либо смерть от инфаркта/инсульта в конечном итоге.
Либо от старости, да. Завязывай нести хуйню.
Аноним 26/12/18 Срд 00:51:17 455525243
>>455524
>Можно бесконечно закачивать натрий/кальций в нейроны
>Завязывай нести хуйню.
Аноним 26/12/18 Срд 00:52:09 455526244
>>455522
Пиздец, как я ненавижу эту присказку, так можно про что угодно сказать, даже про твой любимый учебник по хуйнянейм
Аноним 26/12/18 Срд 00:53:10 455527245
>>455525
Когда нейроны разрушатся, что будет воспринимать боль, олух?
Аноним 26/12/18 Срд 00:54:36 455528246
>>455527
Вот именно, нейроны умрут от сильной и непрекращающейся боли. (С учётом если ещё и блокнуть M, K, опиоидные рецепторы)
Аноним 26/12/18 Срд 13:27:03 455541247
электромагнитный ускоритель

Например, есть пушки которые имея длину в 6-12 метров, с помощью 3 камазов конденсаторов разгоняют тело до нескольких км в секунду.

Вопрос, можно ли от стандартной 16квтч сети создать ускоритель до нескольких км в сек, но разменять конденсаторы на длину ускорителя, то есть ствол будет в несколько сотен метров.

Как примитивно рассчитать если это возможно?
Аноним 26/12/18 Срд 13:35:03 455543248
Вопрос по ОТО: как из постулатов ТО следует замедление времени вблизи массивных тел, или оно из чего-то другого следует?
Аноним 26/12/18 Срд 15:21:27 455550249
15454782032330.jpg (54Кб, 534x507)
534x507
>>455541
> Новости: Двачер с помощью куска водопроводной трубы и силовой электророзетки сбил правительственный самолёт! Польша выразила решительный протест!

Вот ты избавился от трёх Камазов конденсаторов, пусть так (поманяфантазируем).
И добавил три Камаза токопроводящих рельс.
В чём выгода?

Кроме того, без солидного блока конденсаторов всё же никак.

>Как примитивно рассчитать если это возможно?
При некоторых элементарных допущениях, сила, давящая на снаряд, равна магнитной постоянной, делённой на пи, помноженной на квадрат силы тока и на натуральный логарифм отношения диаметра рельс к расстоянию между ними.
Но нахуя?
Аноним 26/12/18 Срд 15:25:20 455552250
Посоветуйте нормальный IQ тест, а то те, которые я находил в интернете, или просят денег или показывают завышенные результаты, чтобы потешить чсв пользователя.
Аноним 26/12/18 Срд 15:29:29 455553251
15457450593830.jpg (18Кб, 300x260)
300x260
Аноним 26/12/18 Срд 15:43:23 455555252
Как доказать, что F=ma
Аноним 26/12/18 Срд 15:54:40 455556253
>>455555
Пентипл туповатый.
Аноним 26/12/18 Срд 15:56:04 455557254
>>455550


>Кроме того, без солидного блока конденсаторов всё же никак.


Почему

>В чём выгода?


Перегрузки, живая бортовая электроника, ресурс.
Аноним 26/12/18 Срд 16:41:21 455561255
Я уже спрашивал как-то, и ответом был не удовлетворен, поэтому задам вопрос еще раз. Почему говно воняет? Нет, не почему оно так интенсивно источает запах, а почему именно воняет? Почему человек воспринимает запах говна как нечто ужасное и мерзкое, а, скажем, запах роз для него -- благоухание?
Аноним 26/12/18 Срд 16:57:56 455562256
>>454356 (OP)
Как изучать естественные науки, как их понять фундаментально?
Для тех, кто слишком стар и туп для этого дерьма

Решил наверстать упущенное, открыл школьные учебники и методички. Читаю раздел элеткромагнетизма: тут эдс, тут фаза. тут ток такой, здесь сила, тут напряжение, здесь формула для полного сопротивления закрытой цепи. здесь формула для плоского конденсатора. здесь формула...тут ток ведет себя так, тут формула.
И НИ СЛОВА о природе всей это лабуды, как я буду запоминать( и зачем?) всю эту огроменную кучу непонятных(с точки зрения приложения) формул, без главного, то есть понимания? Откуда вообще вы берете ток, чтобы обсуждать методы его природы, если я даже не знаю ОТКУДА его берут и КУДА его вставляют(кек). Конечно, я понимаю как васян, что дескать вот энергия распада от ядер на всяких аэс, вот вода нагревается, идет давление, генераторы(щито это?) работают( то есть трутся кек) и вот тебе электричество. Или на небоскребах шпили стоят. Но это ТИПА СЕЙЧАС, а ка как это было изначально? Скаты? Яица трусами натирали? Ловили Зевса за рукоблудием? Че раньше открыли: способы манипуляции электричества или электролиты( то есть суешь метал в химический раствор всяких веществ с незакрытыми слоями электрных орбиталей), то есть батарейки?

Читаю раздел оптики, тонна формул, смотришь на все это и просто впадаешь в уныние. Какие линзы, какие пучки, в жизни этих ваших луп не видал. Ну вот сам очки ношу, че мне с ними делать?

В школах и универчике не было ни объяснений, ни экспериментов. Только бесполезные(потому что я не знаю для чего они, что они, откуда они) формулы, которые совсем не хочется зубрить из-за непонимания.

Читая все это складывается ощущение что как будто твои друзья играют в новую игру и обсуждают всякие скилы и фишечки, а ты просто не шаришь, потому что в ЖИЗНИ блядь ни разу не играл и впервые слышишь.

Неужели нужно быть червем-гением, чтобы во всем этом легко разбираться? Где взять время на изучение всех этих основных аспектов мироздания, этож можно делать всю жизнь? Как тогда жить, лол, монетизировать это знание?

Я просто хз план обучения, вот думаю даже купить какой-нибудь детский набор ака 'собираем свою первую плату сами', авось поможет.
Аноним 26/12/18 Срд 17:27:58 455563257
>>455562
Электродинамика выводится с помощью логики. Простых предположений, проверенных на практике, и одного уравнения. Я серьезно. Только вот для этого надо в матане зашарить.
Аноним 26/12/18 Срд 17:43:40 455565258
>>455563
>практике
Что за практика?
Вот вышел я из дому, солнце херачит мне в лицо, как из этого выведу формулу фокусного расстояния?
Мне обязательно нужно будут вещи с разными средами преломления, то есть для примера я должен буду откопать где-то металлический объект, а если я живу в лесу и у меня с роду не водилось этих ваших металлов?
Или линза, её ведь еще надо выжечь, нужно найти для этого песок.

>матан
Из матана там знание тригонометрии, операции со степенями\дробями и многочленами. То есть это и не матан вовсе, а элементы алгебры.


Че вообще изучают ученые\инженеры, что преподают в универах? Школьный курс это уже овердофига. По скудной практике знаю что половина курсов универа это повторение того же школьного курса. И куда идти дальше?
Вот как люди понимают чего они хотят, если в жизни не занимались этим? Вот хочет человек работать с оптикой, а че это значит, че он делать будет по жизни, как зарабатывать? Вот выучил он формулы, пару эксперементов, освоил по-минимум общие науки, дальше как и куда? Что он может предложить какому-нибудь Никону чтобы попасть в отдел разработок линз, мм?
Аноним 26/12/18 Срд 17:56:07 455566259
>>455565
>Вот хочет человек работать с оптикой, а че это значит
Нет, не хочет.
Аноним 26/12/18 Срд 18:10:52 455567260
>>455562
>>455565
Классический такой диалог:

Q: Как изучать? Хачу ПАНИМАНИЯ!
A: Ну учи матан...
Q: Нихачу матан! Нахуй матан! Хачу ПАНИМАНИЯ!

Матан - это и есть понимание, пердун ты старый.
Аноним 26/12/18 Срд 18:19:28 455569261
>>455567
Я ничего не говорил про матан, ебобо зеленый.
Аноним 26/12/18 Срд 18:22:59 455570262
>>455569
Ну учи матан тогда, чё...
Аноним 26/12/18 Срд 18:26:26 455571263
>>455570
Я знаю матан на своем уровне. Но я не знаю естественные науки.
Аноним 26/12/18 Срд 18:34:00 455572264
>>455571
> Я знаю матан на своем уровне.
Аааа... На "своем" уровне....
> Но я не знаю естественные науки.
Да ну чего ты прибедняешься-то? Ты и естественные науки знаешь, просто на "совем уровне". Т.е. приблизительно на том-же что и матан...
Аноним 26/12/18 Срд 19:40:23 455575265
>>455572
>ааа
ББББ, я проходил матан на первых курсах, когда учился. Я использую матан иногда в своей работе при построении моделей(ака площадь нахождения криволинейной трапеции). Я читал Феймана, где он он на пальцах объясняет модель движения. Че ты от меня хочешь? Я не математик.

>на своем уровне
>на том же что и матан
Нахуй пройди, даун зеленый, нет чтоб просто промолчать, если ничего дельного сказать не можешь. так нет же, будем нести хуйню.
Репорт петуха.
Аноним 26/12/18 Срд 20:23:26 455578266
>>455575
Ебать ты злобный.
Матан в первую очередь это операции с дифференциалами и бесконечно малыми. Вся физика строиться на дифференциальном счисление, причем матан это ентри лвл, который нужно знать в полном понимании, а не на уровне первого курса перед сессией.
Если ты не можешь в физику, значит ты не можешь в матан.
Аноним 26/12/18 Срд 20:24:35 455579267
Двач.. почему ещё не вывели карликовых котов? Кошачие слишком совершенные и поэтому мало подвержены мутациям?
Собак же милипиздрических смогли сделать.
Аноним 26/12/18 Срд 20:34:41 455580268
>>455579
>Двач.. почему ещё не вывели карликовых котов?
Говно без задач. Коты и так довольно маленькие для своего семейства. Впрочем есть породы мелких кошее. Еще меньше делать их, значит сильно понижать их выживаемость.
>Кошачие слишком совершенные и поэтому мало подвержены мутациям?
Что ты в них совершенно нашел? У них довольно много поврежденных генов, которые в принципе им нахуя не сдались. Продолжительность жизни около 10 лет. Скорость мутаций у них такая же как у остальных млекопитающих. Единственный плюс, что у них очень большего генетическое разнообразие в пределах одного вида.

В среднем породы мелких собак соразмерны со среднею кошку.
Аноним 26/12/18 Срд 20:38:08 455581269
>>455580
>Говно без задач.
Сам ты говно без задач. Мелкособак же сделали, а типа они не говно без задач.
>В среднем породы мелких собак соразмерны со среднею кошку.
Именно что мелкособак и среднекошек, а не средне и средне.
Аноним 26/12/18 Срд 21:26:10 455582270
Можно ли силу ветра( сразу скажем турбину классическую) сразу пусть на тепло? То есть без электро энергии
Аноним 26/12/18 Срд 21:28:18 455583271
>>454356 (OP)
Как китайский волчок с точки зрения формул расписать? В гугле не забанили, там где нашел - написано что-то сомнительное, а книжки не находятся.
Аноним 26/12/18 Срд 21:30:00 455584272
>>455582
>сразу пусть на тепло
Нет ничего проще. Да и кпд будет под 100%.
Аноним 26/12/18 Срд 21:38:00 455585273
>>455584


Например?

Мне подумалось, что примитивная турбина вертикальная, и тепло закачивать в тепло аккумулятор из парафина(170 ватт на тонну при 60кг за кило это не шутки)

Хотя и сам придумал, тупо вертикальная и под весом две железки греются, но блядь, сотрется же и как отбирать? гидравликой?
Аноним 26/12/18 Срд 21:38:52 455586274
>>455585


Две железки трутся под землей*
Аноним 26/12/18 Срд 21:39:29 455587275
Аноним 26/12/18 Срд 21:59:59 455589276
>>455575
>я проходил матан на первых курсах, когда учился
>площадь нахождения криволинейной трапеции
В ПТУ учился, да? Помню в седьмом классе прочитал учебник "Алгебра и начала анализа" для ПТУшников. Так с седьмого класса и умею находить площади криволинейных трапеций. А нормального матана ты даже не нюхал.

> Че ты от меня хочешь? Я не математик.

Q: Как изучать? Хачу ПАНИМАНИЯ!
A: Ну учи матан...
Q: Я знаю матан на своем уровне.
A: Ну и знай "на своем уровне"
Q: Че ты от меня хочешь? Я не математик! Нечего сказать ну и молчи!
Аноним 26/12/18 Срд 22:39:05 455590277
>>455562
Фундаментальной наукой занимается не общая физика, а теоретическая. Она тебе расскажет не про ЭДС и катушки, а про то, что электромагнитное поле - это кривизна связности главного расслоения, и про то, как заряд должен существовать из-за калибровочной симметрии, и про то, как оптические лучи описываются бесконечномерными интегралами.
Но для этого надо быть более восприимчивым к советам. Матан знать интуитивно - это не площадь трапеции вывести, а хотя бы уравнение диффузии.
Аноним 26/12/18 Срд 22:55:47 455591278
>>455562
> Для тех, кто слишком стар и туп для этого дерьма


Исходя из опыта - те кто задают такие вопросы - никак, это как музыканты уличные там, просто штырит, такая наклонность, даже не ума а психики.

Можно зазубрить через силу формулы, только словишь депру.


Ну и ещё есть один метод, это, чем то заниматься прикладным, что то конструировать и применять практически, гуглить и тыкать палкой пока не заработает, тогда будет понимание, то есть как с коробкой передач, никто просто так эту хуйню учить не будет.

Вообще каждый пятый теоретик это ебнутый наглухо человек и не надо искать в этом, что то позитивное, лучше ставить прикладные цели, тогда кукуха будет на месте.
Аноним 27/12/18 Чтв 00:02:58 455592279
вопрос не по теме, ну и похуй

У меня в планах прочитать ~200 книг, есть даже списки (в них входит и художка, и научная литература), посмотреть ~100 фильмов и ещё по мелочи пару игр, статьи и т.д. Естественно, этот список будет пополняться со временем.
Внимание, вопрос: за сколько лет я с этим смогу управиться? Как находить время, помимо основной деятельности, ещё и на саморазвитие?
Аноним 27/12/18 Чтв 00:18:31 455593280
>>455592
Ты умрешь раньше, прежде чем сам разберешься в какой либо области.
Смотря какие книги, смотря какие фильмы. Сам понимаешь, что нужны качественные знания, а не количественные.
Научная литература очень специфична, особенно по математике, поскольку там параллельно идет решение задач. В итоге какая-нибудь книжка например общая топология растягивается на несколько лет. Впрочем если у тебя прокачены скилы, ты ее можешь ебнуть залпом за несколько дней. Так же если у тебя есть хороший фундамент знаний, ты даже монографии сможешь за неделю осваивать.
С погромированием полегче, большинству литературы написано чуть ли не для тупых.
Отдельно лингвистика. Тут только зависит от твоих способностей к вниманию, концентрации и памяти, в противном случае сфейлишься.

И вообще, смотри аниме и не выебывайся, саморазвиваться тут захотел, уебок.
Аноним 27/12/18 Чтв 00:42:04 455595281
Эффект-Мейснера.JPG (225Кб, 1425x1900)
1425x1900
Возможно ли масштабировать эффект мейснера для крупных объектов?

Хочу, например, чтобы объект толщиной около полметра парил над над другим на расстоянии от 3-4 метра. Короче как пикрил, но в больших масштабах

Вопросы охлаждения сп, стоимости, энергии, конструкции — опускаем

Есть к этому фундаментальные препятствия?
Аноним 27/12/18 Чтв 01:05:47 455596282
>>455595
Прям фундаментальных нет, но есть закон квадрата куба, материал с определенных масштабах перяет свою прочность и любые резкие изменение вызовут разрушение. Кстати, большинство сверхпроводников довольно хрупкие.
Аноним 27/12/18 Чтв 01:07:46 455597283
>>455580
Хмм, а почему предки собак такие маленькие по сравнению с предками кошек?
Аноним 27/12/18 Чтв 01:16:21 455598284
>>455596
>но есть закон квадрата куба

это про критическое магнитное поле, которое крашит сверхпроводимость?
Аноним 27/12/18 Чтв 01:33:23 455599285
>>455598
Нет. Не совсем.
https://ru.wikipedia.org/wiki/%D0%97%D0%B0%D0%BA%D0%BE%D0%BD_%D0%BA%D0%B2%D0%B0%D0%B4%D1%80%D0%B0%D1%82%D0%B0_%E2%80%94_%D0%BA%D1%83%D0%B1%D0%B0
Суть такова, при увеличение линейных размеров, сила/напряжение растут квадратично, а масса в кубе. В итоге есть пределы на максимальный размер и прочность тел.
В добавок магнитный поток растет квадратично, а плотность энергии в кубе. В итоге начиная с определенных размеров, сверхпроводник тупо разорвет механическим напряжением, создаваемым эффектом вытеснения поля. Или хуже, сверхпроводимость не сможет быть создана из-за слишком больших флуктуаций в больших кристаллах. Впрочем если у нас тело в добавок будет сверхтекучим, то только выиграем!
Аноним 27/12/18 Чтв 01:58:58 455600286
>В итоге есть пределы на максимальный размер и прочность тел.

С помощью чего можно очень примерно посчитать эти пределы, методички есть? Или только эмпирически?

Читал, что так залевитировали элемент гравителескопа весом 5 тонн
Аноним 27/12/18 Чтв 02:04:54 455601287
>>455600
Имперически находим критические параметры, а потом можем легко определить сам предел. Вообще это целиком прикладные задачи и инженегры их разрешают, взять типовую задачу из сопромата про максимальной высоте колоны.
Аноним 27/12/18 Чтв 02:08:58 455602288
>>455601
ну спасибо за науку, пойду к сверхпроводнику прицениваться
Аноним 27/12/18 Чтв 02:34:19 455603289
>>455601
а, еще не могу понять от чего зависит высота левитации, а от чего подъемная сила - и то и то от силы магнитного поля? по раздельности эти параметры нельзя регулировать?

допустим левитирующее тело у меня легкое, интересует только высота
Аноним 27/12/18 Чтв 05:20:41 455605290
>>455600
На мин размер и макс прочность есть Планк.
Аноним 27/12/18 Чтв 07:38:17 455606291
image.png (506Кб, 664x418)
664x418
>>455595
>Возможно ли масштабировать эффект мейснера для крупных объектов?
Аноним 27/12/18 Чтв 08:09:58 455608292
>>455592
1. Строгий распорядок дня + органайзер или хотя бы еженедельная рефлексия "почему я нихуя не успел".
2. Как тут не маневрируй, но фильмы = неэффективная трата времени, за исключением случаев, когда они показывают нечто тебе не досягаемое (животные в африке, туризм в Тибете, хим. опыты, итд). Рассмотри замену на аналогичные видео ютюба или книги. Конечно, если ты хочешь познакомиться с историей кинематографа, то пожалуйста - это интересно, приятно, увлекательнр, но всё ещё бесполезно.
3. Сужай список. Расставь приоритеты. Планируй.
4. Выбирай книги получше. Если в твоём списке, скажем, есть Ландау-Лифшиц, а ты и производную и ту не понимаешь, то рассмотри другие книги - лучше всего на английском, поскольку у них сильнее развита ниша книг для нубов/андерградов.
Аноним 27/12/18 Чтв 10:09:16 455612293
>>455606
маглев на пике работает не на эффекте мейснера
Аноним 27/12/18 Чтв 11:33:22 455614294
>>455612
Про конкретно этот хз. Но на сверхпроводниках маглевы есть.
Аноним 27/12/18 Чтв 12:45:46 455619295
Двоч, оцени идею!!!!!!!!!!!!!!!!!!!!!!!!!!!!!!!!!!!!!!!!!!!!!!!!!!!!



Двигатель стирлинга для зимнего отопления и электроэнергии!!!!




Закапываем головку в землю на глубину в 3 метра, где всегда +5 а часть которая должна быть холодной на поверхности!!!!!!!!!


ХАЛЯВНАЯ ПОСТОЯННАЯ ЭНЕРГИЯ В ЛЮТЫЙ ПИЗДЕЦ ДЕКАБРЬ-ФЕВРАЛЬ!!!!!!!!!!!!!!!!!!!!

ПРОСТО КАК ТРИ КОПЕЙКИ!!!!
Аноним 27/12/18 Чтв 12:54:04 455620296
>>455619


А зимой будет работать в обратную сторону?
Аноним 27/12/18 Чтв 13:01:39 455621297
Аноним 27/12/18 Чтв 13:21:39 455622298
>>455608
Спасибо, вот над строгим распорядком дня уже думаю.
>Органайзер
Блять, как его сделать? В блокноте все не запишешь, заметки на телефоне хаотичны.
Есть какие-нибудь приложения, обязательно с функцией папок, чтобы было удобно организовывать информацию?

>Фильмы бесполезно
Я больше хочу разобраться в самом кинематографе и приправить философией инб4: философия бесполезна. Артхаус там всякий и т.д.

Аноним 27/12/18 Чтв 14:16:27 455623299
>>455619
>где всегда +5
Маленький перепад.
А если ту часть которая тёплая должна быть в доме установить??
Аноним 27/12/18 Чтв 14:48:25 455625300
>>455585
>в тепло аккумулятор из парафина(170 ватт на тонну при 60кг за кило это не шутки)
Зачем, когда есть вода?
Аноним 27/12/18 Чтв 14:59:01 455626301
>>455614
>Но на сверхпроводниках маглевы есть.
Чё прям постоянно жидким азотом охлаждают весь маршрут?
Аноним 27/12/18 Чтв 15:05:14 455627302
>>455623


>Маленький перепад.


Перепад в 30 градусов почти, это много.

Даже мотор не нужен за огромные даллары, я прочитал, что можно тупо воду давить через узкое место и снимать тепло.


Можно топиться за копейки.
Аноним 27/12/18 Чтв 15:05:52 455628303
>>455623


>А если ту часть которая тёплая должна быть в доме установить??


В доме похолодает, это же считай создание силы за счет выравнивания температур.
Аноним 27/12/18 Чтв 15:10:34 455629304
>>455626
В случае применения технологии электродинамического подвеса (EDS) левитация осуществляется при взаимодействии магнитного поля в полотне и поля, создаваемого сверхпроводящими магнитами на борту состава. На базе технологии EDS построены японские поезда JR–Maglev.
Аноним 27/12/18 Чтв 15:12:44 455630305
>>455614
В jr-маглевах действительно используются но для каких-то вторичных целей, не мейснеровских
Аноним 27/12/18 Чтв 15:13:38 455633306
>>455619
>Двигатель стирлинга для зимнего отопления и электроэнергии!!!!
>
Лучше можно. Берешь такой двигатель к холодильнику, и питаешь сам холодильник от полученной энергии.
Аноним 27/12/18 Чтв 15:15:23 455634307
>>455627
>Можно топиться за копейки.
Ты типа решил затраллить sci как b? Ну тут постов по типу "ДА ТЫ ТУПОЙ" с пояснениями почему ты не прав не будет. Поэтому ты не сможешь покормиться.
Аноним 27/12/18 Чтв 15:15:47 455635308
>>455633


В итоге холодильник превращается в обогреватель.
Аноним 27/12/18 Чтв 15:18:54 455637309
>>455634


А чего тупого? можно механически снимать киловатты на разнице температур в грунте и на поверхности всю зиму, без сотен тыщ рублей.
Аноним 27/12/18 Чтв 15:20:53 455638310
>>455629
A very strong magnetic field is required to levitate a train. The JR–Maglev trains have superconducting magnetic coils, but the JR–Maglev levitation is not due to the Meissner effect.
Аноним 27/12/18 Чтв 15:22:33 455639311
>>455637
Энергия от этого двигателя, на самом деле энергия дров, которые сжигают, чтобы тебе батареи греть.
мимо-КЭП
Аноним 27/12/18 Чтв 15:25:04 455640312
>>455639
Читай с начала, пиздоглазый.
Никакие дрова стирлингу не нужны, на ютубе есть примеры как он в комнате(+20) на ладоне работает.
Аноним 27/12/18 Чтв 15:43:39 455641313
>>455640
Хорошо, попробую с другой стороны.
Вот у тебя в лесу стоит дом, внутри дома +5 под домом +5. С чего ты начнешь?
Аноним 27/12/18 Чтв 15:49:24 455642314
>>455641
С улицы где -25, даунич
Аноним 27/12/18 Чтв 15:58:43 455643315
>>455642
Хорошо. На улице -25 и в доме -25. Дальше что?
Аноним 27/12/18 Чтв 16:04:06 455644316
>>455643
Ты траллить тупостью пытаешься?
Аноним 27/12/18 Чтв 16:08:55 455645317
>>455644
Я просто жду конкретное описание процесса. Но ты как-то не очень можешь воспроизвести свои мысли в текст.
Аноним 27/12/18 Чтв 16:16:48 455646318
>>455645
Прочитай ещё разок свои сообщения и найди ещё одно место, в котором может быть какая-то температура.
Аноним 27/12/18 Чтв 18:22:50 455647319
>>455619
>Двигатель стирлинга для зимнего отопления и электроэнергии!!!!
У Стирлинга во 1х още плохое отношение мощности к материалоёмкости как для земных условий, во 2х для сколь-либо приемлемого КПД при земных температурах - в качестве рабочего тела нужон водород при 60-70 атмосферах и температура горячей части под 1000 С, в 3х КПД начинает сильно расти при смещении рабочего интервала температур в сторону 0 К, т.е. это двигатель для далёких плонет где есть много халявного льда при -150 -200 С.
Но если ты начнёшь выкачивать самопальным Стирлингом тепло на из коммиблока улицу за КПД в 1-2% - то я тебя найду, выдам пиздюлей и всего обосру.
Аноним 27/12/18 Чтв 20:54:07 455654320
>>455647
>тепло на из коммиблока улицу
Да выдохни ты, пусть делает, по телевизору покажут. Принципиально новый генератор Сычёва. Президентский грант.
Аноним 27/12/18 Чтв 23:12:44 455656321
FractalBroccoli.jpg (125Кб, 1024x768)
1024x768
>>454356 (OP)
1. Посоны, почему на больших глубинах океана рыбы не потеряли зрение с течением эволюции? Всякие пауки и прочая мерзость в пещерах теряют зрение, а те же удильщики - нет.

2. Если на планетах типа Энцелада уществует рачковая жизнь, может ли у подобных существ развиться зрение и биолюминесценция, если они никогда не воспринимали свет?
Аноним 28/12/18 Птн 02:12:45 455658322
images.jpg (9Кб, 252x200)
252x200
>>455619
>Двоч, оцени идею
а может просто тепловой насос сделаешь для отопления, как куча народа до тебя?
Аноним 28/12/18 Птн 02:15:39 455659323
>>455656
потому что на глубине присутствует свечение вызванное распадом калия40 достаточное для их глаз, плюс биолюминесценция на каждом шагу
Аноним 28/12/18 Птн 02:24:01 455660324
Кажется, я нашел беспрецедентный слет катющиковцев и прочих альтернативно одаренных на тему ОТО, можно бесконечно лоллировать с комментариев https://www.youtube.com/watch?v=4f3H03QTUsc
Аноним 28/12/18 Птн 05:18:17 455661325
>>455660
Прямо гнездо шизоидов всех мастей.
Аноним 28/12/18 Птн 07:48:11 455663326
>>455140
Он падает, но постоянно промахивается. Как Земля и Луна.
Аноним 28/12/18 Птн 08:04:32 455664327
1.jpg (45Кб, 1280x720)
1280x720
>>454356 (OP)
Сап, ученые мужи. Поясните неофиту, где у вас тут загон для любителей альтернативной энергетики и энергетики вообще? С кем тут можно подискутировать о холодном ядерном синтезе, графеновых аккумуляторах, ветряках, солнечных панелях, электромобилях и прочем?
Аноним 28/12/18 Птн 08:15:32 455665328
>>455664
>холодном ядерном синтезе
Математически вроде доказали, что нириально.
Аноним 28/12/18 Птн 08:32:26 455666329
>>455665
Ок, тогда о управляемом термояде. Или даже просто о ядерной энергетике. Я, признаться, чуть не поехал, когда осознал, через какие костыли работают АЭС. Они же за почти 100 лет практически не изменились. Ну, автоматики разве что больше стало, а технология та же: ведро с радиоактивным варевом, вокруг которого намотан змеевик с водой; вода нагревается, пар свищет и вращает турбину. Ну это же смех, да и только. КПД 30% - пушка просто. Когда мы нормальные АЭС выдумаем, а не это позорище?
Аноним 28/12/18 Птн 09:05:13 455667330
>>455666
> Ну это же смех, да и только
Изучи вопрос конкретней. Начни с термодинамики, чтобы понять, какое теоретически возможное КПД можно сделать.
Да и потом, ты неправильно описал процесс. Надо так:
"В виде теплоносителя, используется редчайшее в своих свойствах вещество - оксид протия, идеально сочетающее в себе как нужные для теплоносителя свойства так и экологичность с ценой."
Аноним 28/12/18 Птн 09:21:52 455668331
>>455666
>Я, признаться, чуть не поехал, когда осознал, через какие костыли работают АЭС. Они же за почти 100 лет практически не изменились. Ну, автоматики разве что больше стало, а технология та же: ведро с радиоактивным варевом, вокруг которого намотан змеевик с водой; вода нагревается, пар свищет и вращает турбину.
А ты уже узнал что термояд собираются использовать точно также?
Аноним 28/12/18 Птн 09:23:43 455669332
>>455666
>Ну это же смех, да и только.
И ты конечно же уже готов предложить что-то другое, принципиально иное?
Аноним 28/12/18 Птн 11:47:47 455670333
>>455667
но ведь оксид дейтерия лучше!
Аноним 28/12/18 Птн 14:29:10 455671334
>>454356 (OP)
почему человек не разделился на виды когда начал осваивать новые территории?
Аноним 28/12/18 Птн 15:39:23 455672335
>>455671
Разделился
Но за такие взгляды после известных событий 20 века - тупо бан
Аноним 28/12/18 Птн 15:42:29 455673336
>>455672
Если образовался новый вид , то люди не могли бы спариватьсясекс друг с другом
Аноним 28/12/18 Птн 15:42:55 455675337
>>455671
И у животных есть "подвид". Раса ближе к этому по своей сути. На вид разделится не могли, ибо времени мало прошло.
Аноним 28/12/18 Птн 15:46:30 455676338
>>455675
скорее всего могли бы ещё. гибриды существуют. и бывают и стабильные. неандертальцы же могли с кроманьонцами смешиваться. Но чем дальше расхождение, тем всё более нестабильные гибриды, бесплодные, а после и вовсе не выходит. Но для такого полноценного размежевания тысяч 400 лет нужно.
Аноним 28/12/18 Птн 15:46:46 455677339
Аноним 28/12/18 Птн 15:47:59 455678340
>>455673
>не могли бы спариваться
Птички могут.

https://royalsocietypublishing.org/doi/full/10.1098/rsbl.2018.0557
>early generation female hybrid between a golden-winged warbler (Vermivora chrysoptera) and a blue-winged warbler (V. cyanoptera) went on to mate and successfully reproduce with a chestnut-sided warbler (Setophaga pensylvanica).
>wood-warblers in general have remained genetically compatible long after they evolved major phenotypic differences.

Прямо как у людей, которые остались генетически совместимы после того как наэволвили значительные фенотипические различия. Только вот птичек почему-то в такой же ситуации считают разными (хоть и близко родственными) видами, а чоловиков - нет.
Аноним 28/12/18 Птн 15:51:01 455679341
>>455678
Так фенотип не влияет на способность скрещивания
Аноним 28/12/18 Птн 15:53:26 455680342
>>455679
>фенотип не влияет на способность скрещивания
>>455673
>Если образовался новый вид , то люди не могли бы спариваться

Птички с разным фенотипом, не влияющим на способность скрещивания, помименованы учеными как разные виды.

Приматы с разным фенотипом, не влияющим на способность скрещивания, считаются (((учёными))) одним видом.

Ферштейн?
Аноним 28/12/18 Птн 15:53:47 455681343
>>455676
>Но для такого полноценного размежевания тысяч 400 лет нужно.
Первым покинул Африку и заселил Евразию Homo erectus, миграции которого начались около 2 млн лет назад. Разве этого времени мало?
Аноним 28/12/18 Птн 15:58:38 455682344
>>455681
Ну так межвидовая борьба. После пришли неандертальцы и вломили им пиздюлей, те и вымерли. Человек такая тварь, что могла жить везде, а значит и расселялась везде, поэтому выжить мог только один вид. Ибо изолироваться друг от друга не могли и жить спокойно. Хотя по теориями некоторые эректусы пережили и неандертальцев на разных островах, но и до них по итогу добрались. Ну и катаклизмы разные поспособствовали, на островах это считай конец популяции.
Аноним 28/12/18 Птн 16:01:58 455683345
>>455682
>После пришли неандертальцы и вломили им пиздюлей, те и вымерли.
Первые люди с чертами протонеандертальца существовали в Европе ещё 350—600 тысяч лет назад, последние неандертальцы жили около 40 тыс. лет назад. И ты хочешь сказать что прямоходящим не хватило 1 миллиона лет чтобы перейти в разные виды?
Аноним 28/12/18 Птн 16:06:21 455684346
>>455678
Только вот беда фенотип==генотип.
У человечества в целом довольно маленькое генетическое разнообразие. Генофонд всей неафриканской популяции людей в среднем одинаковый, даже генофонд иранцев и филиппинцев различаются только на уровне статической погрешности. Это говорит от том, что в популяции курсируют одни и те же гены, только с разными частотами, собственно разность частот генов у различных популяции и выдается за расы. В среднем же выходит, что людей даже на подвид нельзя разделить.
Далее уже у всех людей довольно многогенов нуклеотидных последовательностей, которые отсутствуют у близжайщего вида- шимпанзе. Это уже явный намек, что все люди один. А мутации в этих последовательностях позволяет нам даже определять относительную родство и восстанавливать общего предка. Такого даже у собак нет.
Аноним 28/12/18 Птн 16:06:39 455685347
>>455683
Почему не могли, могли и переходили. Неандертальцы и кроманьёнцы это разные виды. Но выжил по итогу лишь один. Ибо конкуренция жёсткая межвидовая, я же написал об этом. Иначе и быть не могло. Потому что в те времена не редко соседи из одного вида воспринимались как добыча на уровне с мамонтами, а уж из другого и подавно. Съели неандертальцев кроманьёнцы, как и до них неандертальцы съели эректусов и всех остальных, кто там мог бы быть.

Аноним 28/12/18 Птн 16:09:02 455686348
>>455683
Ереткусы просто не успели оставить альтернативных потомков, кроме 3,5 групп макак.
Аноним 28/12/18 Птн 16:10:13 455687349
>>455684
Почему тогда птички с одним набором генов в той же ситуации - разные виды, а люди - один?
Аноним 28/12/18 Птн 16:33:58 455688350
>>455687
Таксонопроблемы.
Раньше просто не было адекватного способа установления родства.
Впрочем есть довольно забавная ведь, близкородственные виды генетические различаются сильнее, чем не родственные виды.
Аноним 28/12/18 Птн 17:16:59 455689351
Сцач, вопрос не совсем по науке.
В чём ты рисуешь стереометрию? Автокад — оверкилл, но в целом рассматривается (не он сам, а свободные клоны, офк).
Аноним 28/12/18 Птн 20:42:41 455692352
>>454356 (OP)
Какие механизмы помогают найти мутантную клетку в организме и почему не все мутации можно обнаружить?
Аноним 28/12/18 Птн 21:22:49 455696353
В гугле не нашел


Двигатель стиргинга работает в обратную сторону если холод и тепло поменять местами?
Аноним 29/12/18 Суб 00:46:18 455697354
>>455692
Скажем так, мутантные клетки обнаруживаются не потому что они мутантные, а потому что они ведут себя ебануто. Между клетками есть особые способы передачи сигналов, а добавок на поверхности клеток есть уникальный набор сигнальных белков. Что-то из этого не правильно, значит наш клетка впала ересь и она враг. Впрочем иногда бывают сбои связанные с экспрессией генов или повреждение белков, из-за чего клетка начинает вести не правильно, при этом последовательность днк не нарушена.
Аноним 29/12/18 Суб 00:46:20 455698355
>>455689
В скетчапе построй, там и стилизация под иллюстрацию имеется
Аноним 29/12/18 Суб 09:30:44 455704356
>>455687
Птички, про которых ты говоришь, в природе не скрещиваются - поведение слишком различается. А вот если их в одной клетке держать, тогда да, могут. Однако генетический признак вида не единственный же, вы почему-то все об этом забываете
Аноним 29/12/18 Суб 09:32:28 455706357
>>455704
какие еще есть признаки у вида?
Аноним 29/12/18 Суб 09:55:16 455707358
>>455706
Морфологический, Биохимический, поведенческий, экологический, географический
Аноним 29/12/18 Суб 10:08:06 455708359
>>455707
а подробнее не распишешь?
Аноним 29/12/18 Суб 10:33:51 455713360
evolutionofmode[...].jpg (120Кб, 1050x700)
1050x700
>>455706
В антропологии, например, вид человека (и расу, и этнос группы) выделяют по анализу измерений черепа и скелета или анализу зубов. В генетике - по количеству накопленных изменений генотипа.
Подвид неандертальца, к примеру, выделен потому, что размеры костей не попадают в размах изменений костей всех сапиенсов на планете. Не может, например, сапиенс иметь ключицу такой длины при таком росте.
Более того, все неандертальцы кучкуются на статистических диаграммах, потому виды просто-напросто очевидны при таком анализе.
Аноним 29/12/18 Суб 13:52:42 455722361
15458954768970.jpg (34Кб, 420x673)
420x673
Какого черта я имею походняк как полуДЦП, если на то, чтобы научиться писать левой каллиграфическим почерком у меня ушло 4 месяца ленивых занятий на похуй, а еще я обладаю возможностью овладевать идеальным произношением любого слова любого языка за 1-3 попытки? Почему походка не совершенствуется блядждьб?
Аноним 29/12/18 Суб 14:00:02 455723362
>>455722
Ах да! У меня же со спиной проблемы, тут скорее всего спинной мозг поврежден. Нужно было всего секунду подумать.
Аноним 29/12/18 Суб 18:09:17 455737363
Можно ли делать так (при V1=V2, зная значение P1, P2, T1):
P1V1 = nRT1; nR = P1V1 / T1
P2V2 = nRT2

T2 = P2V2/nR; T2 = P2V2 x T1 / P1V1;
T2 = P2 x T1 / P1
?
Аноним 29/12/18 Суб 18:57:23 455738364
>>455737
Ты сейчас спрашиваешь можно ли пользоваться математикой?
Аноним 29/12/18 Суб 19:05:33 455739365
>>455738
Просто у меня какие-то дикие числа в результате получаются.
Аноним 29/12/18 Суб 19:09:58 455741366
>>455739
С размерностями не тупи. Правильные:
Па, K, м3
Аноним 29/12/18 Суб 19:15:23 455742367
>>455741
Да все в сях:
P1 = 7370Pa
T1 = 313.15K
V1 = 1M

P2 = 101325
T2 = ?
V2 = V1 = 1M

T2 = 101325 x 313.15 / 7370
T2 = 4305K

Это 4032 по Цельсию.

Бля, тупанул. n1 != n2.
Аноним 29/12/18 Суб 19:16:06 455743368
>>455742
Или равны. Запутался.
Аноним 29/12/18 Суб 19:19:00 455744369
>>455743
Короче, объем газа в открытом сосуде нагревается до Т2, давление атмосферное. Дальше сосуд запечатывается и остужается до Т1 и в результате должно получиться давление P1.
Все таки n1 = n2.
Аноним 29/12/18 Суб 19:23:49 455745370
>>455737
хули ты развел, pV/T = const. для одинаковый газов пропорциальное уравнение p1V1/T1 = p2V2/T2.
Аноним 29/12/18 Суб 19:23:52 455746371
Бля че за ебанистика Про комментарии у двача?
Аноним 29/12/18 Суб 23:00:44 455769372
>>454356 (OP)
Хотелось бы конечно более-менее умный ответ.
Ну там известно, можно в википедии прочитать, как взрывают, современные ядерные бомбы. Добиваются, чтобы ударная волна от обыкновенного взрыва, долбанула шарик из плутония со всех сторон одновременно. Он от такого удара сжимается-уплотняется, ну и дальше пошло поехало, но дело не в этом.
Наверняка отрабатывали это все на каких-нибудь стальных-медных-свинцовых шарах. И интересно, что происходило с ними от таких экстремальных воздействий?
Нагуглить у меня ни на русском ни на английском не получилось
Аноним 30/12/18 Вск 11:24:47 455776373
15445558857530.png (128Кб, 408x485)
408x485
>>454544
Хуя нибамбит то как
Аноним 30/12/18 Вск 13:17:50 455778374
15460693391640.jpg (380Кб, 1666x944)
1666x944
>>454544
> а мы вычисляем протеины и живём вечно молодыми
Аноним 30/12/18 Вск 14:01:22 455780375
C5854C8C7.jpg (136Кб, 1024x1638)
1024x1638
>>454544
>живём вечно молодыми
Тот же самый флуктуативно не стабильный энтропийно зависимый копрокал говнины или по простому хрень какая то с оттягиванием неизбежного костылями, щитова только абсолютная неуязвимость чтоб можно было хреналиард чисел грема лет в сингулярности черной дыры сычевать и думоть при этом позновая тру недвойственность, остальное нищитова и не бесконечность нихрена!
Аноним 30/12/18 Вск 14:11:16 455781376
Без названия.jpeg (12Кб, 225x225)
225x225
>>455780
>чтоб можно было хреналиард чисел грема лет в сингулярности черной дыры сычевать и думоть
Сычуешь такой время стремящееся к бесконечности внутри чёрной дыры, а у самого на обдумОвание вышло в итоге количество времени, стремящееся к нулю.
Аноним 30/12/18 Вск 14:17:18 455782377
15335645529370.png (1672Кб, 1500x1000)
1500x1000
148567474517975[...].jpg (65Кб, 400x196)
400x196
>>455781
Зато таки вечен в реально неуязвимой капсуле.
Аноним 30/12/18 Вск 14:22:13 455783378
15442040113800.jpg (212Кб, 1218x1015)
1218x1015
149916546415615[...].jpg (24Кб, 300x300)
300x300
>>455782
Да но из - за стазиса времени у тебя интеллекта как у супа частиц в кварк глюонной плазме, так как думать не можешь без времени, типо не очень то по бессмертному выходит.
Аноним 30/12/18 Вск 15:08:13 455784379
15461206412470.png (946Кб, 784x894)
784x894
>>455783
>Да но из - за стазиса времени у тебя интеллекта как у супа частиц в кварк глюонной плазме, так как думать не можешь без времени, типо не очень то по бессмертному выходит.
Выход есть -- >>455143 .
Обмазываемся чёрными дырами вокруг себя, получаем вечную (с точностью до испарения чд) неуязвимую (с точностью до распада ложного вакуума) пространственно-временную капсулу. Можно думать о протеинах.
Аноним 30/12/18 Вск 22:00:25 455789380
"температура кипения" означает и то что при этой же температуре вещество может/будет находиться в виде газа?
Аноним 30/12/18 Вск 22:11:41 455791381
>>455789
Да, только чтобы из одного состояния в другое перейти, надо Latent heat приложить.
Аноним 30/12/18 Вск 22:28:34 455792382
>>455789
Нет, вещество может находится в виде газа даже вблизи абсолютного нуля. Вообще постоянно идет два противоположенных процесса - конденсации и испарения. Баланс задается давлением насыщенных паров, т.е такое давление газа, при котором его испаряется столько же сколько обратно конденсируется. Это хуйня называется двухфазное состояние.
Температура кипения это же такая температура, при котором давления насыщенного пара равно внешнему. Обычно под температурой кипения понимается такая температура при которой давление насыщенного пара равно одной атмосфере.
Впрочем есть критическая температура, при котором вещество не сможет обратно сконденсировать в жидкость, и так же критическое давление, выше этого давления разрушается равновесии испарения/конденсации и вещество не может существовать в виде двух фазной системы.
Аноним 30/12/18 Вск 22:37:54 455793383
>>455792
>Впрочем есть критическая температура, при котором вещество не сможет обратно сконденсировать в жидкость
Она типа просто немного выше температуры кипения, или какая-то определенная?
Аноним 30/12/18 Вск 22:45:55 455795384
>>455793
Критическая температура вполне определенна и специфична для вещества. Да она всегда выше температуры кипения, но между ними разница может быть различна. Например, для ртути критическая температура 1476 , а для воды 373 градусов по Цельсию.
Аноним 30/12/18 Вск 22:51:35 455796385
>>455795
Интересно, всем спасибо за ответы
Аноним 30/12/18 Вск 22:55:53 455797386
Если сжиженный(от сильного охлаждения) газ поместить в замкнутый объём, равный занимаемому им как жидокстью, а затем дать нагреться до ну, он останется жидкостью, или станет газом с большим давлением?
Аноним 31/12/18 Пнд 00:16:12 455805387
>>455797
Смотря какой газ и в какой объем.
Если нагреть выше определенной температуры, то он станет сверхкритической жидкостью (свойства как у жидкости, только без поверхностных эффектов), потом совсем станет почти как газ.
Аноним 31/12/18 Пнд 00:52:21 455806388
>>455805
Просто я смотрел видосик про эволюцию ядерных бобм, и не врубился зачем было городить сложные и здоровенные установки для охлаждения для поддержания водорода в виде жидкости (типа по расчётам это лучшее из состоянии для этого), если можно было просто закачать его под большим давлением.
Потом подумал что может при нормальной температуре для этого требуется такое давление, что ни один материал не выдержит, но посмотрел диаграмму состояние-и для этого нужно не более 10МПа.
Аноним 31/12/18 Пнд 01:05:11 455807389
>>455806
Водород хранят жидким не потому что требуются большие давления, а потому что он может под больших давлениях растворятся в материале и таким образом утекать. Особенно этим грешат металлы.
Однако позже открыли, что металлогидриды прям на ура могут растворять в себе водород, причем в одном объеме растворится водорода больше, нежели его жидкая форма.
Аноним 31/12/18 Пнд 01:15:09 455808390
>>455807
А, похоже потому что больше чем жидкость не закачаешь, а потом пару минут, немного водорода через стенки убежало, и уже не жидкость, в газ перешёл, а им очень хотелось чтобы это вот прям гарантированно жидкость была, для чистоты эксперимента..
Хотя можно ж было сделать компенсатор утечек, типа пружинного например.. ну да ладно
Аноним 31/12/18 Пнд 01:23:57 455809391
>>455807
>Однако позже открыли, что металлогидриды прям на ура могут растворять в себе водород, причем в одном объеме растворится водорода больше, нежели его жидкая форма.
Кстати, хоть и любая материя по большей части "пустая" при н.у., но это кажется каким-то нереальным
Аноним 31/12/18 Пнд 02:31:00 455814392
>>455809
Даже в обычной воде водорода на единицу объема больше, нежели чем в жидком водороде.
И вообще, материя нихуя не "пустая", об этом говорит все разнообразие химических связей.
Водород же протон и электрон. Электроны хорошо отжимают многие атомы, проблемы только с массивным протоном, которого особо не подвигаешь. Хотя есть класс химических веществ, которые могут "утилизировать" свободный протон, от же аммиак.
Аноним 31/12/18 Пнд 11:04:59 455829393
>>455814
Какой-то у тебя странный взгляд на размеры.
Аноним 31/12/18 Пнд 12:01:51 455833394
>>455814
>Даже в обычной воде водорода на единицу объема больше, нежели чем в жидком водороде.
Ну так это химическое соединение, а растворение его в этих металлогидридах, что, тоже химическое соединение?
Аноним 31/12/18 Пнд 13:07:07 455835395
>>455833
Химическая связь понятие очень растяжимое. А водород вполне себе может образовывать металлические связи, ибо существует металлический водород.
Короче типа сплавы.
Аноним 31/12/18 Пнд 15:29:04 455840396
>>455835
Что несёт, что несёт..
Аноним 31/12/18 Пнд 15:34:12 455841397
>>455840
Что тебя смутило, что водород может быть металлическим или что водород с некими металлами образует металлическую связь?
Аноним 31/12/18 Пнд 15:37:59 455842398
>>455841
Меня смутило что ты не разделяешь вещества образующиеся в результате химической реакции и растворы/сплавы.
Растяжимое понятие у него блять.
Аноним 31/12/18 Пнд 15:46:21 455843399
>>455842
Интерметаллиды это вещества или сплавы?
Водородная связь это химическая связь? А если мы говорим про [HF2]-?
Идеальна ли классификация таких понятий как "химическая связь"?
Клатраты газов, в которых только межмол взаимодействия, не имеют химических связей, но имеют конкретные фазы, четкую кристаллическую решетку и свои термодинамические характеристики.
Где проходит граница что является химическим соединением а что не является, когда абсолютно все взаимодействия в химии, от ионных в кристаллах до ван-дер-ваальсовых в жидком гелии имеют одинаковую природу?
Аноним 31/12/18 Пнд 16:15:45 455848400
Аноним 01/01/19 Втр 04:58:32 455887401
>>454379
Какой-то непонятный ответ.

Давайте я, другой анон, более конкретную ситуацию обрисую:

Допустим, Марс находится в 5 световых минутах от Земли.
Допустим, у меня есть космический корабль, способный доставить меня с Земли на Марс за 1 секунду (да-да, знаю, ну а вдруг? Вдруг когда-нибудь изъебнутся и изобретут такой?).

В 10:00:00 я отправился с Земли на Марс, в 10:00:01 прибыл на Марс и потом увидел, что в 10:01:00 включили лазер, направленный на Землю. В 10:02:00 я отправился на Землю, в 10:02:01 прибыл на Землю и сообщил, что в 10:06:00 земляне увидят свет лазера с Марса.

НУ И ЧТО? Разве я что-то нарушил? Я же не сказал им будущее, лазер-то уже включился - это прошлое, просто на момент, когда я это сказал, они еще не увидели эффект от включения. Но они не могут изменить прошлое.

Не вижу тут никаких парадоксов. Правильно же, их нет?
Аноним 01/01/19 Втр 05:04:15 455888402
>>454490
> для существования вселенной нужно 3е лицо в виде бога
Почему-то существование вселенной ты считаешь нужным попытаться объяснить, а вот существование бога не утруждаешься
Аноним 01/01/19 Втр 09:51:13 455891403
>>455656
Бамп второму вопросу.
Аноним 01/01/19 Втр 12:37:35 455894404
>>455891
Да всё возможно, йопта
Аноним 01/01/19 Втр 12:39:29 455895405
>>455658
Ну и что бле он сделает в геотермально неактивном регионе?
Аноним 01/01/19 Втр 13:39:00 455896406
>>455887
Потому что существует система отсчета, где события меняются местами.
Мне сейчас лениво придумывать события, которые бы это показали, может позже приду и нарисую.
Аноним 01/01/19 Втр 15:47:50 455900407
Аноним 01/01/19 Втр 15:55:11 455901408
>>455896
Да уж, интересно было бы
Аноним 01/01/19 Втр 16:01:44 455902409
>>455114
Зависит еще, попала ли твоя нога под колесо (или в дырку под машиной) сразу же, мне кажется. Если да, то ты никуда и не полетишь, сразу под машину
Аноним 01/01/19 Втр 17:01:25 455906410
IMG201901011628[...].jpg (47Кб, 677x934)
677x934
Двач, поясни
Есть коробка в которой летают сталкиваясь абсолютно упруго друг с другом частицы. (Из коробки через то йобаокошечко слева они вылететь не могут)
Через йобаокошечко 1 слева, в коробку в направлении прямо вправо запускаем много частиц.
Места их ПЕРВОГО соударения со стенками коробки будут примерно соответствовать графику на стенке слева?
Если:
1)Запускаемые частицы при соударении не оказывают никакого влияни на частицы в коробке(имеют нулевую массу).
2)Имеют схожую с частицами в коробке массу.
Аноним 01/01/19 Втр 17:30:07 455907411
>>455906
>Запускаемые частицы при соударении не оказывают никакого влияния на частицы в коробке(имеют нулевую массу).
Нулевая масса не предполагает нулевого импульса.
Но вообще да, картинка будет какая-то такая. Теорвером можно проверить, там типа на каждый отрезок пути, будет вероятность столкнуться с чем-то и в сложении выйдет нормальное распределение.
Аноним 01/01/19 Втр 18:04:49 455909412
>>455907
Т.е. отсюда и берётся прямолинейность распространения света в среде?
Аноним 01/01/19 Втр 18:18:35 455911413
image.jpeg (662Кб, 1080x580)
1080x580
Аноним 01/01/19 Втр 18:37:31 455912414
>>455909
да, кратчайшие расстояния дают наибольший вклад. Хотя отдельные кванты могут лететь куда угодно.
Аноним 01/01/19 Втр 18:41:21 455913415
>>455909
>отсюда и берётся прямолинейность распространения света в среде?
Со светом всё несколько сложнее. Он всё же квантовый.
Аноним 01/01/19 Втр 19:20:15 455915416
>>455913
>Со светом всё несколько сложнее. Он всё же квантовый.
Но в целом всё так, или всё совсем не так?
Аноним 01/01/19 Втр 19:24:51 455916417
>>455915
Если грубо говорить, то так.
Аноним 01/01/19 Втр 19:49:17 455917418
>>455916
А если точно?
Под столкновениями кстати подразумевалось поглощение и переизлучение в случайном направлении. Хм.., что впрочем совсем не то что и столкновение, но в среднем результат должен быть таким же..(или нет?)
Аноним 01/01/19 Втр 20:01:18 455918419
>>455912
>>455913
Это ж получается зная скорость света в среде можно всякую информацию о этой среде вычислять.. А хотя не, только какой-то собирательный образ плотности и температуры.

Если взять луч света видимой длины волны, пропущенный через воздух при нормальных условиях, то при какой "толщине" среды количество фотонов столкнувшихся хоть раз с молекулой газа будет стремиться к 100%?
Хотя воздух лучше заменить каким-то односоставны газом или идеальным, хз
Аноним 01/01/19 Втр 20:04:48 455919420
>>455911
Блен, как бы уйти вообще от электронов и передавать энергию чисто полем
Аноним 01/01/19 Втр 20:06:48 455920421
>>455918
>акой-то собирательный образ плотности и температуры.
Там зависимость от электронного строения есть.
>Если взять луч света видимой длины волны, пропущенный через воздух при нормальных условиях, то при какой "толщине" среды количество фотонов столкнувшихся хоть раз с молекулой газа будет стремиться к 100%?
>Хотя воздух лучше заменить каким-то односоставны газом или идеальным, хз
См. закон Бугера-Ламберта-Бера, по экспоненте затухает интенсивность.
Аноним 01/01/19 Втр 20:07:57 455921422
>>455919
Ничего тебе не мешает. Бери оптоволокно и веди. Или лазером свети куда тебе надо.
Аноним 01/01/19 Втр 21:17:21 455923423
>>455921
*электрическим полем конечно же подразумевалось
Там же о электронах говорится.
Аноним 01/01/19 Втр 21:29:52 455924424
>>455923
Я вообще не знаю чё ты хочешь. Сформулируй мысль.
Аноним 01/01/19 Втр 21:37:37 455925425
>>455561
Наверное чтобы закопать/избавиться от него поскорее, чтобы микробы из него не отравляли человека
Аноним 01/01/19 Втр 21:49:42 455927426
>>455592
На скорости 3х смотреть фильмы, а лучше 5-минутные версии этих фильмов на ютубе. Книги в кратком пересказе для шкальников, или 5-минутные версии экранизаций этих книг на ютубе, или на крайняк аудиоверсии на 3х скорости. И побольше энергетических напитков!
Аноним 01/01/19 Втр 22:00:55 455928427
>>455673
Почему же я тогда в /b/ видел кучу видосов, где песики спариваются с женщинами?
Аноним 01/01/19 Втр 22:03:41 455929428
>>455928
критерий вида это потомство у внуков, а так и бабуины могут тебя отодрать
Аноним 01/01/19 Втр 22:33:05 455930429
Аноним 01/01/19 Втр 22:34:02 455931430
>>455929
Наверное подобным образом и спид появился - бабуины отодрали кого-то в африке
Аноним 01/01/19 Втр 22:37:11 455932431
>>455781
Можно во все комплюктерные игры переиграть не торопясь, и все секреты в них найти. У меня лично занять себя всегда есть чем
Аноним 01/01/19 Втр 22:41:05 455933432
>>455784
Тоже пикча топовая
Аноним 01/01/19 Втр 23:40:48 455937433
Сап, двач. Я хуй знает по адресу или нет.

Расскажите, как опубликоваться в каком-нибудь научном журнале (в приоритете, конечно, из перечня ВАК и тд)? Есть какой-то гайд или типа того? Интересует в частности вопрос, как строить работу с преподом, а так же процесс взаимодействия с журналом.
Аноним 01/01/19 Втр 23:45:16 455938434
>>455937
В любой журнал отправляешь свою статью, рецензенты думают принимать её или нет. Конечно они смотрят на фамилию, на твою историю публикаций и на фамилии твоих соавторов.
Аноним 02/01/19 Срд 00:53:50 455943435
Поясните по теории относительности, грав замедление времени вытекает из релятивистского замедления или там из чего-то другого это получается? от чего ведется логическая цепочка?
Аноним 02/01/19 Срд 01:51:31 455946436
>>455943
Если ты будешь сидеть в ящике, а я этот ящик буду ускорять с ускорением g. То ты внутри не сможешь сказать покоится ли ящик на Земле, либо его кто-то ускоряет в космосе. Законы физики в двух этих случаях внутри ящика должны быть идентичны, а значит гравитация действует аналогично ускорению. Отсюда начинают плясать, а дальше там уже математика возникает с метрическим тензором.
Аноним 02/01/19 Срд 03:56:28 455956437
>>455943
Для объяснение ТО и как ее эффекты проявляются нужно объяснять дифференциальную геометрию, тензорный анализ, а потом еще вариационные исчисления.
Готов ли ты к этому говну для объяснения логической цепочки?
И нет, грав замедление времени это самостоятельный эффект, причем он больше геометрический природы, нежели чем просто из преобразований относительности.
Аноним 02/01/19 Срд 04:04:28 455957438
Аноним 02/01/19 Срд 05:42:05 455958439
>>455957 Как нахождение человека в движущемся вагоне влияет на то что для него передняя сторона не будет убегать а задняя догонять свет идущий и середины?
Аноним 02/01/19 Срд 07:17:00 455959440
>>454356 (OP)
После какого срока беременности появление сиамских близнецов более не возможно?
Аноним 02/01/19 Срд 07:20:20 455960441
Аноним 02/01/19 Срд 07:21:02 455961442
Аноним 02/01/19 Срд 09:57:00 455967443
>>454545
Сук, как же проорал
Ему нужно движение кнн создавать - калькуляторы не нужны
Аноним 02/01/19 Срд 10:38:43 455969444
>>455958
> Как нахождение человека в движущемся вагоне влияет
Никак. Человек тут ни при чём.
Аноним 02/01/19 Срд 12:53:21 455972445
>>455969
Как-то уклончиво получилось.
Аноним 02/01/19 Срд 15:23:23 455982446
Аноним 02/01/19 Срд 15:55:21 455985447
image.jpeg (684Кб, 2208x1242)
2208x1242
image.jpeg (1016Кб, 2208x1242)
2208x1242
image.jpeg (290Кб, 2208x567)
2208x567
Читаю прошлый тред, и просто ору с этого поста
Аноним 03/01/19 Чтв 01:38:38 456015448
image.jpeg (163Кб, 480x689)
480x689
Тоже ору
Аноним 03/01/19 Чтв 06:45:01 456017449
1546487094581.png (500Кб, 1080x2280)
1080x2280
Аноним 03/01/19 Чтв 13:10:23 456024450
>>455960
Частицы – стоячие волны полей, виртуальные частицы – флуктуации.
Аноним 03/01/19 Чтв 13:52:28 456025451
>>456024
>виртуальные частицы – флуктуации
Виртуальные частицы, это когда математикой начинаешь пользоваться и теорию возмущений подключаешь.
Аноним 03/01/19 Чтв 13:56:27 456026452
>>456024
Частицы могут имеет непрерывный спектр, т.е могут быть бегущей волной, епта. Более того, все свободные частицы представляют как волновой пакет, где фаза явно не зависима.
Виртуальные частицы это не флуктуации, это блядь поправки в амплитудах рассеивания, поскольку материя у нас не частичная, а таки полевая.
Аноним 03/01/19 Чтв 14:12:33 456027453
Если говорят что в вакууме постоянно возникают пары частица-античастица и бесследно незаметно аннигилируют, то почему говорят о получении энергии из антиматерии по формуле e=mc^2, т.е. полностью?
Есть разные виды аннигиляции?
Аноним 03/01/19 Чтв 14:39:55 456028454
image.png (22Кб, 220x264)
220x264
image.png (28Кб, 220x198)
220x198
>>456027
>Если говорят что в вакууме постоянно возникают пары частица-античастица и бесследно незаметно аннигилируют
То это гоев пытаются наебать на доллары. Знаешь, есть короче в школьной физике такой способ искать потенциал поля, "метод изображений", там дорисовывают просто еще заряды, чтобы в проводниках поле стало 0. Так вот, при этом все понимают, что этих зарядов дорисованных нет, это просто математический трюк. А ВОТ НАУЧПОПЕРЫ РЕАЛЬНО ПРО ВИРТУАЛЬНЫЕ ЧАСТИЦЫ ЗАТИРАЮТ, хотя там одного уровня уловка.
Аноним 03/01/19 Чтв 14:48:05 456029455
maxresdefault ([...].jpg (143Кб, 1920x1080)
1920x1080
Отражает ли уровень памяти здорового человека его возможности к познанию и осмыслению? Опустим прецеденты травмированных головой умных людей, они могут потерять что-то одно (воображение, возможность качественно излагать мысль), а память зачастую страдает меньше всего, ибо отделы за неё отвечающие сидят глубоко. Вопрос в том, может ли с виду типикал тупой быдлан васян с соской пиваса обладать очень хорошей памятью и мне лучше бы перед ним ничем важным не светить, иначе запомнит и додумается воспользоваться?
Аноним 03/01/19 Чтв 15:19:13 456030456
Поцоны, я почитал срачи на тему катющика и у меня возник тупой вопрос: когда говорят, что искривляется пространство-время, то имеется ввиду "4-х мерный объект пространство-время", а не видимое 3-х мерное пространство? Оно остается визуально прежним, но там замедляется время и меняются траектории движения тел?
И второй вопрос: если магнитное поле это релятивистское следствие движение электрического, то почему оно именно в таком виде появляется (перпендикулярно)?
Аноним 03/01/19 Чтв 15:36:20 456031457
>>456027
Хуй его знает откуда это пошло, скорее от научпоперов.
Короче так, когда начали пилить теорию квантового поля, выяснилось, что фундаментально частицы сами по себе не могут существовать, есть только набор непрерывных полей. Но частичность материи мы таки наблюдаем, да плюс есть старая проблема с самодействием. Это решили элегантным способом, теперь материя проявляет частичные свойства только через операторы уничтожение-рождения частиц. В итоге имеем, что частицы это некоторое спектральное разложение полей. Все взаимодействие частиц, это преобразование полей между собой или в самих себя.
Скажу по другому, материя существует не как частицы, а как суперпозиция всех мысленных полей, частицы лезут тогда, когда мы пытаемся запихнуть квантовые поля в статические рамки.

>почему говорят о получении энергии из антиматерии по формуле e=mc^2, т.е. полностью?
Тебя смущает энергия покоя частиц? При аннигиляция мы получаем не энергию в чистом виде, просто поля переходят другие поля, могут даже в сами себя. Из-за фундаментальных свойств полей энергия покоя тоже считается энергией и ее нельзя в никуда деть, вот и приходится ее впихивает в дополнительную энергию, если результат аннигиляции будет две безмассовые частицы.

>Нет, аннигиляции?
Нет, с хуя должны быть разные?
Аноним 03/01/19 Чтв 15:42:28 456032458
image.png (79Кб, 518x389)
518x389
>>456030
Если ты гнешь пространство-время, то у тебя и трехмерное гнется.
>если магнитное поле это релятивистское следствие движение электрического, то почему оно именно в таком виде появляется (перпендикулярно)?
Потому что когда ты начинаешь выводить законы электродинамики, то пользуешься векторным анализом, и у тебя просто это вылазит из векторного произведения.
Аноним 03/01/19 Чтв 15:44:19 456033459
image.png (80Кб, 490x437)
490x437
Аноним 03/01/19 Чтв 15:51:06 456035460
>>456029
Есть под памятью подразумевается способность образовать новые связи между различными нейронами, и при этом адекватно снижать старые связи, то да. Это напрямую определяет на все когнитивные способности.
>типикал тупой быдлан васян с соской пиваса обладать очень хорошей памятью
Это же иллюзия, епта. Скорее всего из-за того, что у быдлана узок кругозор и набор проблем, которых надо решать, он предпочитает пользоваться отработанными паттернами проведениями, которые могут самоусиливают и ускоряют время реакции, в итоге у него простые вещи легко получается и он запоминает конкретные вещи.
Уровень когнитивных способностей у быдлана будет повыше, чем у рядового посетителя досок с картинками в интернете.
Аноним 03/01/19 Чтв 15:52:24 456036461
>>456032
>у тебя просто это вылазит из векторного произведения
а есть более физическое объяснение, помимо того, что это автоматом получается в мат аппарате?
Аноним 03/01/19 Чтв 15:55:23 456037462
>>456036
Обязательно должно быть. Надо просто подумать немного.
Аноним 03/01/19 Чтв 16:00:34 456038463
>>456036
Это хуйня в общем виде называется дифференциальные формы.
Физически это означает, что ЭМ поле у нас это 4-вектор, который меняется в соответствие с преобразований 4-координат.
"Магнитная" часть это просто огрызок преобразований на 3-мернго пространство.
Аноним 03/01/19 Чтв 16:15:13 456039464
>>456035
>снижать старые
Ох уж эти мантры. Что-то уровня "фоторгафическая память будет мешать перед глазами, хорошо, что у меня её нет и я вообще не знаю что это".
Мои воспоминания неподвластны времени. Не представляю, как бы я жил, будучи животным с памятью на три дня. Я бы с ума сошел еще когда в универе учился и приходилось запоминать по 300 страниц текста 10 калибри. Я за неделю его только один раз мог осилить, а если бы еще и перечитывать пришлось, охх...
Аноним 03/01/19 Чтв 16:26:37 456044465
>>456032
>Если ты гнешь пространство-время, то у тебя и трехмерное гнется.
А тут есть какой-то порядок/причина-следствие ? То есть изменяется пространство, а в следствии время, или наоборот, или нет такого?
Аноним 03/01/19 Чтв 16:30:57 456045466
>>456044
>А тут есть какой-то порядок/причина-следствие ?
Эм... пространство это кусок "пространства-времени". Если ты будешь деформировать обычное пространство, то какой "порядок/причина" деформации плоскости XУ в этом случае?
Аноним 03/01/19 Чтв 16:44:45 456046467
>>455121
Чёрных дыр не существует.
Аноним 03/01/19 Чтв 17:58:48 456052468
>>456045
То есть искривление пространства-времени - это искажение и того и другого в 3-х мерном, а не хитрая проекция, где изменение одного видно, а другого нет? И когда объект притягивается к массивному телу, он сам же не видит что его траектория изменилась, он для себя прямо движется.
Аноним 03/01/19 Чтв 18:12:01 456053469
15464472776130.png (360Кб, 500x677)
500x677
>>456046
> Я сама дрочь Кипа Торна, пруфов не будет. Поверьте, в Лебедь X-1 всё не так однозначно, никто не хочет сингулярности!
Аноним 03/01/19 Чтв 18:25:14 456054470
>>456046
ну белые то есть?
Аноним 03/01/19 Чтв 19:16:54 456056471
>>456039
>Мои воспоминания неподвластны времени. Не представляю, как бы я жил, будучи животным с памятью на три дня.
>Я бы с ума сошел еще когда в универе учился и приходилось запоминать по 300 страниц текста 10 калибри.
И твой манявыйгрышь в спортлото не помешал тебе быть опущенным говноедом-хуесосм, как иронично.
Аноним 03/01/19 Чтв 21:33:13 456059472
>>456052
> и того и другого в 3-х мерном
Трехмерное пространство, лишь проекция четырехмерного.
>когда объект притягивается к массивному телу, он сам же не видит что его траектория изменилась
Ну как сказать. Всё-таки свет и частоту меняет и направление, думаю заметить, что что-то с пространством не так, в сильных полях можно, но я не спец, я никогда задачи из ОТО не решал.
Аноним 03/01/19 Чтв 23:08:28 456062473
maxresdefault ([...].jpg (204Кб, 1920x1080)
1920x1080
Аноним 04/01/19 Птн 14:52:59 456091474
t.jpg (43Кб, 500x300)
500x300
>>454356 (OP)
1) Чем лагранжевое изложение классической механики лучше ньютоновоского? Зачем принцип наименьшего действия вместо более интуитивных законов ньютона если в итоге всё равно приходим к тем же уравнениям? Чем ценен этот феноменологический принцип именно с точки зрения философии, почему вдруг природа должна занулять какой-то там функционал, как это пришло в голову?
2) Насколько стохастический анализ вообще в финансах что-то даёт? Ну т.е. не в теории, а в оценке инструментов на реальном рынке. Или black box стратегии чисто на машинном обучении уже намного эффективнее? В частности, интересует блэкшолтц, насколько эта модель правильно оценивает опционы на реальном рынке?
3) Топологический дата анализ это просто модная игрушка? Или он реально полезнее классических статистических методов? Почему при построении баркод диаграммы мы натягиваем на метрический граф именно двумерный комлпекс и считаем только 0,1 и 2-е гомологии?
Аноним 04/01/19 Птн 15:57:17 456093475
>>456091
1) Тем чем лагранжевое изложение классической механики лучше ньютоновоского. Принцип наименьшего действия вместо более интуитивных законов ньютона в итоге всё равно приходим к тем же уравнениям?. Этот феноменологический принцип ценен именно с точки зрения философии, природа должна занулять какой-то там функционал.
2) Стохастический анализ вообще в финансах что-то даёт. Ну т.е. не в теории, а в оценке инструментов на реальном рынке. black box стратегии чисто на машинном обучении уже намного эффективнее. Модель правильно оценивает опционы на реальном рынке.
3) Топологический дата анализ это просто модная игрушка. Он реально полезнее классических статистических методов? При построении баркод диаграммы мы натягиваем на метрический граф именно двумерный комлпекс и считаем только 0,1 и 2-е гомологии.
Аноним 04/01/19 Птн 22:47:19 456109476
>>454356 (OP)
Когда одноклеточный организм делится, то каков статус материнской клетки? Они считается погибшей или что?
Аноним 04/01/19 Птн 22:53:20 456111477
Двач, внутри чёрных дыр, за горизонтом событий, только масса первичной чёрной дыры, возникшей непосредственно во время коллапса материи, а вся остальная масса распределена/размазана ДО горизонта и скрыта от глаз красным смещением?

Двач, внутри чёрной дыры масса, стремящаяся к нулю?
Аноним 04/01/19 Птн 22:55:29 456112478
Можно ли взять нейросеть, натренировать ее на какую-то задачу, а потом каким-то образом превратить её в сложный алгоритм? Или нейросеть это черный ящик?
Аноним 04/01/19 Птн 22:58:53 456113479
>>456112
>а потом каким-то образом
Каким-то можно.
Аноним 04/01/19 Птн 23:06:39 456114480
>>456113
Можно взять нейросеть и дать ей в качестве данных натренированные нейросети и алгоритмы которые они должны были повторить. И потом напустить ее на саму себя. Профит?
Аноним 04/01/19 Птн 23:11:22 456115481
>>456114
>И потом напустить ее на саму себя. Профит?
Что это значит и в чём профит?
Аноним 04/01/19 Птн 23:15:47 456116482
>>456091
Лагранжевое изложение во-первых легче решает многие задачи быстрее, так как Лагранжиан составить намного проще. Во-вторых туда входят обобщенные координаты, то есть это могут быть как xyz так и любые другие даже криволинейные. Во-вторых изложение Лагранжа связывает закон сохранения импульса с с однородностью пространства, закон сохранения момента с изотропностью пространства, а закон сохранения энергии выводит как закон однородности времени. В-третьих, красиво получается выводить СТО, электродинамику и ОТО. Например лангражиан свободной частицы в СТО это просто -mc. А если есть электромагнитное поле то -mc-e/cA . Очень лаконичные и простые на вид формы, при этом если добавить третий член связанный с действием самого поля, то можно вывести математически все 4 уравнения Максвелла причем и в трехмерной и если хочешь в четырехмерном виде.
Короче, чисто теоретически, принцип наименьшего действия имеет большую ценность, так как обобщает многие физические законы.
Аноним 04/01/19 Птн 23:16:24 456117483
>>456116
два раза написал во-вторых, но ты понел
Аноним 05/01/19 Суб 02:50:06 456125484
1546645799907.png (581Кб, 1080x2280)
1080x2280
Absorbance это в чем? В теле человека или где?
Аноним 05/01/19 Суб 09:52:59 456128485
>>456091
>Чем лагранжевое изложение классической механики лучше ньютоновоского?
Позволяет описывать динамику системы в любых подходящих координатах.
>Зачем принцип наименьшего действия вместо более интуитивных законов ньютона если в итоге всё равно приходим к тем же уравнениям?
За выражения типа "интуитивно понятный" и прочее впору вешать. А так, как уже было сказано, лагранжев формализм оказывается очень полезным, позволяет много чего обобщить. Из него же потом появились гамильтонов формализм, вакономная механика и прочее.
>как это пришло в голову
Всё началось с принципа Ферма.
> Насколько стохастический анализ вообще в финансах что-то даёт? Ну т.е. не в теории, а в оценке инструментов на реальном рынке. Или black box стратегии чисто на машинном обучении уже намного эффективнее?
Ключевая особенность нашего времени - это святая вера в компьютер. Особенно это выражается у относительно юного поколения. А так в целом, это вещи эквивалентные по своей эффективности.
Аноним 05/01/19 Суб 10:11:57 456129486
>>456093
[Оффтоп]
Не с мехмата часом?
Аноним 05/01/19 Суб 13:19:56 456134487
>>456128
>А так в целом, это вещи эквивалентные по своей эффективности.
Типа одинаково неэффективные?
Аноним 05/01/19 Суб 13:45:00 456135488
>>456134
>Типа одинаково неэффективные?
То что лежит в открытом доступе - неэффективно. То что есть у всяких аналитиков - вполне может быть эффективно, но это скрыто за семью коммерческими тайнами. Что вполне разумно.
Аноним 05/01/19 Суб 16:09:40 456140489
>>456125
нижняя ось это длина волны
Аноним 05/01/19 Суб 16:18:26 456141490
>>456115
Не знаю, это же тред глупых вопросов. Попробую еще раз сформулировать.

У нас есть определенные алгоритмы. Ну, предположим, сортировки чисел пузырьком, мерджем, etc или чего-нибудь такое. Мы генерируем кучу случайный последовательностей, потом сортируем их. Это таблицу мы используем для обучения нейросетей. У нас выходит семейство нейросетей A, которые учились сортировать.
Потом возьмем другой алгоритм, допустим, поиск максимума. Опять генерируем материал, учим. Выходит семейство Б. И так далее.
Теперь мы берем нейросеть R. В качестве данных предоставляем ей закодированные определенным образом (например в специальном упрощенном ЯП) алгоритмы и дамп обученной нейросети из семейств А,Б,В итд. В итоге нейросеть R должна учиться распознавать алгоритм по дампу обученной нейросети.
А теперь вопрос: на каком этапе возникнет затруднение и что будет, если нейросети R скормить свой собстевнный дамп?
Аноним 05/01/19 Суб 16:51:38 456144491
Аноним 05/01/19 Суб 17:34:39 456146492
>>456141
если нет исключений в работе алгоритмов 1 2 3 4, т.е. все сводится к какой- либо линейной функции,то проблем нет. Если функция нелинейна, много "если так, то результат умножаем на -1" и т.п., то функция охуеет и будет как тот робот пылесос, который хотели обучить ездить без столкновений, а он начал ездить задом. Ну, в принципе возможно, если в примерах у каждой функции будут рассмотрены все минимумы, максимумы и исключения и все их взаимные произведения., т.е. фактически почти все значения функции последней. т.е. суть такой нейросети - в том чтобы обработав большой объем данных она смогла не хранить его, а выдавать нужное значение из новых данных. И если какие-либо значения не будут рассмотрены, то последняя нейросеть не разберется в них. Я тебе говорю как человек, который работает с нейросетью. Она ничего новго не придумывает, если примеров нету, она выдает ахинею.
Аноним 05/01/19 Суб 18:54:32 456151493
Аноним 05/01/19 Суб 20:00:23 456154494
>>456151
>внутри чёрной дыры масса, стремящаяся к нулю?
Почему? Логика непонятна. В гипотетических ЧД масса распределена не равномерно, в центре - сингулярность, вокруг все что туда падает, оно менее плотное, ЧД не комок сверхплотной материи, окруженный горизонтом, а экстремальная область пространства. В теории, чем больше ЧД, тем меньше плотность и приливные силы вблизи горизонта событий, и туда можно провалиться не заметив этого.
Аноним 05/01/19 Суб 21:22:27 456158495
>>456154
>и туда можно провалиться не заметив этого.
А про замедление времени ты конечно же забыл.
Аноним 05/01/19 Суб 21:28:34 456160496
15463414562290.png (271Кб, 601x302)
601x302
>>456111
> Двач, внутри чёрных дыр, за горизонтом событий, только масса первичной чёрной дыры, возникшей непосредственно во время коллапса материи, а вся остальная масса распределена/размазана ДО горизонта и скрыта от глаз красным смещением?
Попробуй тогда ответить на вопрос, почему гравитационные волны при слиянии ЧД фиксировали.
> Двач, внутри чёрной дыры масса, стремящаяся к нулю?
Лолнет, но направление твоей мысли забавное. В том смысле, что можно попробовать создать и такую теорию, и посмотреть, что выйдет.

>>456154
>В гипотетических ЧД масса распределена не равномерно, в центре - сингулярность, вокруг все что туда падает, оно менее плотное,
Эй-эй, полегче в этой ситуации! Я тебе сейчас керровские ЧД припомню с отрицательными расстояниями и кольцевыми сингулярностями!
Мимо-проходил
Аноним 05/01/19 Суб 21:38:26 456164497
>>456160
>Попробуй тогда ответить на вопрос, почему гравитационные волны при слиянии ЧД фиксировали.
Масса-то большая, просто не внутри горизонта а снаружи.
Ты мне лучше скажи как их фиксировали, если масса внутри него.
Я там себе выдумывал разное(до того как вопрос про массу стремящуюся к нулю внутри чд задал) типа гравитационное поле из которого невозможно выбраться ничему(т.к. даже свет не может) одной чёрной дыры, в момент слияния со второй как-то компенсирует поле второй, и чему-то таки удаётся вырваться.

>Лолнет, но направление твоей мысли забавное.
Что забавного может быть в самом логичном варианте рассмотрения этой йобы?
Свет не может вырваться?=>"гравитация" такая сильная что время остановленно полностью. Нет течения времени-зайти за горизонт невозможно, можно только приближаться бесконечно близко за период времени стремящийся к бесконечности.
Аноним 05/01/19 Суб 21:48:18 456170498
>>456160
ну понятно, что имеется ввиду простой вариант, а не вращающаяся
Аноним 05/01/19 Суб 22:00:39 456172499
>>456158
А разве падающий объект увидит это? Со стороны то очевидно, что время для него остановится.
Аноним 05/01/19 Суб 22:04:40 456173500
>>456172
>А разве падающий объект увидит это?
А ты уверен что ему будет что смотреть?
Вселенная тоже как бэ "со стороны", времени её жизни может не хватить на падение.
Аноним 05/01/19 Суб 22:06:04 456174501
15454380270840.jpg (6Кб, 226x178)
226x178
>>456164
> Ты мне лучше скажи как их фиксировали, если масса внутри него.
Слушай, почитай лучше сам, а?
https://habr.com/post/406435/

> типа гравитационное поле из которого невозможно выбраться ничему(т.к. даже свет не может) одной чёрной дыры, в момент слияния со второй как-то компенсирует поле второй, и чему-то таки удаётся вырваться.
Эм, нет, разве что они обе висят где-то в Римановом пространстве бесконечной кривизны. Но это больше из области фэнтези.

> Что забавного может быть в самом логичном варианте рассмотрения этой йобы?
Не воспринимай в отрицательной коннотации. Я именно что выразил поддержку развитию подобного направления мысли. Если оно, конечно, будет поддержано математическим аппаратом.

> Нет течения времени-зайти за горизонт невозможно, можно только приближаться бесконечно близко за период времени стремящийся к бесконечности.
Да я понял.
Только учти, что время квантуется же. Как и пространство. И Ахилессом тут не стать.
А так ты огненный файерволл, кажется, переизобрёл, только вид сбоку (в спойлере пояснение).
Аноним 05/01/19 Суб 22:14:29 456176502
>>456174
>Только учти, что время квантуется же. Как и пространство. И Ахилессом тут не стать.
А как квантование может помешать, если время для каждого следующего шага через квант пространства увеличивается, а не уменьшается или остаётся прежним?

Да, я то нормально воспринял, то именно вопрос а не упрёк был.
Аноним 05/01/19 Суб 22:46:05 456177503
А я все ещё не понимаю. Вот смотрите: мы находимся в корабле, двигающемся со скоростью n досветовая мы светим фонарём из переднего конца в задний. Датчик, стоящий в заднем конце будет фиксировать увеличенную скорость же, ведь он двигается навстречу свету. Так было бы логично, но препод по физике в универе меня чуть не побила за этот вопрос мы знаем, что она абсолютная и все равно будет равна с. Как так? Почему датчик не за фиксирует скорость с+n?
Аноним 05/01/19 Суб 22:47:09 456178504
>>456177
Сраный т9 вставил пробел, а меня теперь коробит от этого
Аноним 05/01/19 Суб 22:53:30 456180505
>>456177
>Как так?
Второй постулат теории относительности.
Аноним 05/01/19 Суб 22:55:46 456181506
>>456180
Более подробно объясни, плез
Аноним 05/01/19 Суб 23:02:26 456183507
>>456181
Принцип постоянства скорости света: свет в любой инерциальной системе отсчета распространяется с одной и той же скоростью, независимо от того, испущен он движущимся или неподвижным источником.
Аноним 05/01/19 Суб 23:05:05 456184508
>>456183
Спасибо, брат, теперь понял
Аноним 05/01/19 Суб 23:59:15 456188509
>>456177
Короче так, суть ТО в том, что есть совокупность частиц, и есть некоторое(подразумевается ЭМ, но подойдет другие) поле, через которое частицы взаимодействуют с друг другом. Поля у нас распространяются с конечной скоростью, из-за этого мы должны учитывать еще и время, т.е есть некоторая "задержка" перед которой частицы "чувствуют" друг друга. Однако из этого следует другое сильное утверждение, из-за того, что частицы могут "чувствовать" друг друга только через поле, то и свойства поля определяет какой мир будет видеть частица.
Главная принцип ТО это собственно принцип относительности - все физические законы в инерциальных системах отсчета одинаковые. Если мы это перефразируем, то частицы "чувствуют" поле всегда одинаковые, т.е для них оно всегда будет распространяться с определенной постоянной скоростью.
В итоге имеем, что у нас мир мы можем наблюдать только через поле, скорость которого конечна и поле для всех "наблюдателей" ведет по отношений к ним самим одинаково. Если тело движется относительно наблюдателя, и оно посылает сигнал - деформирует поле, то поскольку скорость поля у нас определенная, оно будет просто искажаться/изменяться сигнал - (эффект доплера) вместо увеличение скорости света.
Аноним 06/01/19 Вск 00:00:35 456189510
Жобрый вечер, двачик.

Хочу научиться проектировать реактивные двигатели самостоятельно. Звучит смешно, но я закончил универ по похожей специальности, могу делать статические и динамические расчёты на прочность, расчёты газов и тд, и всё все то же на соответствующем софте, так что считаю это возможным.

Так вот, возможно в треде совершенно случайно окажется человек, который знаком с тематикой и может мне посоветовать литературу? Нагуглить что-то годное по теме нереально, даже простейшие концептуальные расчёты вроде какой тяги достаточно для подъёма такото-то планера самолёта, или количество воздуха продуваемое через двигатель при такой-то тяге, и тп. Ну вы поняли, был бы очень благодарен.
Аноним 06/01/19 Вск 00:48:59 456192511
Какая степень окисления у антиводорода?
Аноним 06/01/19 Вск 00:50:22 456193512
Я тут размышлял о смысле бытия и возник вопрос: закон тяготения и закон кулона идентичны, в электро-магнетизме есть магнитная составляющая, появляющаяся при движении, есть ли что-то похожее в гравитации? И наоборот - что-то схожее с невесомостью есть в других полях?
Аноним 06/01/19 Вск 02:27:20 456195513
>>456193
Ну таки да, в стационарном виде закон кулана и гравитация одинаковы, а еще они являются полноценными полями заданными с одинаковыми преобразованиями. Да, у гравитации есть аналог магнитный составляющей.
Впрочем схожесть в приближении гравитации и Электромагнетизма вызвано свойствами пространства и его симметрией. На этом сходство заканчивается. Главная различия в том, что ЭМ поле калибровочное - линейное, а ОТО вообще говоря нет и при достаточно сильных полях становится очень нелинейным.
Второе различие начинается в неэквивалентных математических формулировках. Если очень грубо, то ЭМ поле задается как структура поверх пространства-времени, а в ОТО гравитация задается как структура пространство-временем. Короче действие в лагранжиане задают различными способами.
Аноним 06/01/19 Вск 02:33:44 456197514
Аноним 06/01/19 Вск 02:42:40 456198515
>>456195
>Да, у гравитации есть аналог магнитный составляющей
и это?
>Если очень грубо, то ЭМ поле задается как структура поверх пространства-времени, а в ОТО гравитация задается как структура пространство-временем.
То есть можно сказать, что гравитация - самое фундаментальное поле, а остальные поля в него вложены и неизбежно взаимодействуют с ним?
Аноним 06/01/19 Вск 03:54:05 456202516
>>456198
>и это?
Списочек в конце.
https://ru.wikipedia.org/wiki/%D0%93%D1%80%D0%B0%D0%B2%D0%B8%D1%82%D0%BE%D0%BC%D0%B0%D0%B3%D0%BD%D0%B5%D1%82%D0%B8%D0%B7%D0%BC
Впрочем они не являются прям аналогичными "магнитными" как у ЭМ поля, но являются схожими из общих соображений.
>То есть можно сказать, что гравитация - самое фундаментальное поле, а остальные поля в него вложены и неизбежно взаимодействуют с ним?
В некотором смысле да, в некотором смысле нет.
Все дело в том, что мы считаем полем. ЭМ и прочие поля это грубо говоря некоторая функция от координат, которую можно дифференцировать, а еще обладает ряд свойств вроде преобразование относительно себя. Гравитационное поле нельзя так определить, для этого используются методы дифференциальной геометрии. В некотором смысле гравитационное поле это совокупность набора функций, которые зависят не только от координат, но еще зависят от направления вдоль которого мы будем дифференцировать их. И чтоб они ложились в полевую теорию, нужно исходит из других соображений, неужели другие полевые теории. Короче говоря, гравитация в полевой теории стоит особняком из-за других математических методов, и гравитационное поле называют полем, потому что нет других адекватных определений. С другой стороны, другие поля тоже можно представить через геометрические представления, но там успеха мы не имеем, потому что эта нам ничего не дает для обобщения и в предсказательной силе, только все усложняет.
Вкладывать ОТО в ЭМ принципе можно, и даже пытались, но грянула квантовая теория с другим формализмом поля, где гравитации не нашлось места.
Максон-патисон 06/01/19 Вск 05:17:52 456204517
Есть ли способ изменить цвет глаз? И есть ли тред по этой теме?
Аноним 06/01/19 Вск 16:16:04 456224518
Аноним 06/01/19 Вск 16:24:57 456225519
>>456204
Чем тебя ношение соответствующих контактных линз не устраивает?
Аноним 06/01/19 Вск 21:02:05 456245520
>>456204
Йозеф Менгеле в треде, все в газовую камеру
Аноним 08/01/19 Втр 00:42:19 456342521
>>454543
Съебись, зелёный
Аноним 10/01/19 Чтв 11:25:13 456478522
В чем биологическое и эволюционное назначение чувства ностальгии? Есть ли что-то подобное у животных?

Что мне сразу приходит на ум: защитным механизм психики, разгрузка приятными воспоминаниями о прошлом, в котором всё было хорошо — стимул для выживания.
Аноним 10/01/19 Чтв 15:08:05 456491523
>>456478
не у всего есть назначения. многое у человека просто фича, следствие сознания и множество других факторов психики. и прочертить тут просто одну линию "вот это нужно для этого" нельзя.
Настройки X
Ответить в тред X
15000 [S]
Макс объем: 40Mб, макс кол-во файлов: 4
Кликни/брось файл/ctrl-v
Стикеры X
Топ тредов
Избранное